Docsity
Docsity

Prepare for your exams
Prepare for your exams

Study with the several resources on Docsity


Earn points to download
Earn points to download

Earn points by helping other students or get them with a premium plan


Guidelines and tips
Guidelines and tips

Well Child Care and Preventive Pediatric Health Care Guidelines, Exams of Pediatrics

Insights into the american academy of pediatrics recommendations for well child care, focusing on illness prevention, nutrition, and child development. It also discusses the impact of physical activity and sports on children's health, including those with asthma and down syndrome. The document also touches on ethical and behavioral issues, medication administration, and pediatric drug studies.

Typology: Exams

2023/2024

Available from 05/30/2024

Advancewrighter
Advancewrighter 🇺🇸

4

(10)

3K documents

1 / 134

Toggle sidebar

Related documents


Partial preview of the text

Download Well Child Care and Preventive Pediatric Health Care Guidelines and more Exams Pediatrics in PDF only on Docsity! Burns Pediatric Final Exam Study Guide Questions with Verified Answers. 1. The primary care pediatric nurse practitioner understands that a major child health outcome associated With worldwide climate change is: A. cost of living. B. education. C. nutrition. D. pollution. - Correct answer ANS: C There is growing evidence that climate change is having a dramatic effect on food crops that leads to food Distribution issues and food insecurity among families. 2. The primary care pediatric nurse practitioner understands that, to achieve the greatest world- wide Reduction in child mortality from pneumonia and diarrhea, which intervention is most effective? a. Antibiotics b. Optimal nutrition c. Vaccinations d. Water purification - Correct answer ANS: C Rotavirus is the most common cause of diarrhea globally and Strep pneumonia is the leading cause of Pneumonia, and together these are the leading infectious causes of childhood morbidity and mortality Globally. Both are vaccine-preventable diseases. Antibiotics to treat pneumonia, optimal nutrition, and Clean water all help to reduce morbidity and mortality, but vaccination prevents the diseases from Occurring. 3. When providing well child care for an infant in the first year of life, the primary care pediatric nurse Practitioner is adhering to the most recent American Academy of Pediatrics Recommendations for Preventive Pediatric Health Care guidelines by: A. focusing less on development and more on illness prevention and nutrition. B. following guidelines established by the Bright Futures publication. C. scheduling well-baby visits to coincide with key developmental milestones. D. seeing the infant at ages 2, 4, 6, and 12 months when immunizations are due. - Correct answer ANS: C In the most recent AAP Recommendations for Preventive Pediatric Health Care, there is a greater Emphasis on behavioral and developmental issues and a recommendation that well child care be based on Child and family development rather than the periodicity of immunization schedules. This will require a Revision of the current recommendations in Bright Futures. 4. Which is true about the health status of children in the United States? a. Globalism has relatively little impact on child health measures in the U.S. b. Obesity rates among 2- to 5-year-olds have shown a recent significant decrease. c. The rate of household poverty is lower than in other economically developed nations. d. Young children who attend preschool or day care have higher food insecurity. - Correct answer ANS: B Obesity rates are a major concern for child health in the U.S. but recently have stabilized in the rate of Increase and have declined among 2- to 5-year-olds between 2004 and 2013. Globalism has an increasing Effect on child health in the U.S. The rate of household poverty in the U.S. is higher than in other Economically developed nations. Young children who attend preschool or day care have lower food Insecurity. 5. Which region globally has the highest infant mortality rate? P a g e 1 | 134 a. Indonesia B. Southern Asia c. Sub-Saharan Africa d. Syria - Correct answer ANS: C Although Sub-Saharan Africa and Southern Asia together account for 81% of the infant mortality rate Globally, Sub-Saharan Africa has the highest infant mortality rate in the world. 1. The parent of a toddler is concerned that the child may have autism. The primary care pediatric nurse Practitioner completes a Modified Checklist for Autism in Toddlers (M-CHAT) tool, which indicates Several areas of concern. What will the nurse practitioner do? a. Administer a Childhood Autism Rating Scale (CARS) in the clinic. b. Consult a specialist to determine appropriate early intervention strategies. c. Refer the child to a behavioral specialist for further evaluation. d. Tell the parent that this result indicates that the child has autism. - Correct answer ANS: C The M-CHAT is a screening tool and is useful for detecting behaviors that may indicate autism. This Instrument has been found to have acceptable sensitivity, specificity, and significant positive predictive Value. If these behaviors are detected, the PNP should refer the child to a specialist for further assessment, Using more diagnostic tools. The CARS may be used but requires specialty training and proper Credentials. Until the diagnosis is determined, strategies for intervention are not discussed. The M-CHAT Is a screening tool and is not diagnostic. 2. The mother of a newborn tells the primary care pediatric nurse practitioner that she is worried that her Child will develop allergies and asthma. Which tool will the nurse practitioner use to evaluate this? Risk? A. Three-generation pedigree b. Review of systems c. Genogram d. Eco map - Correct answer ANS: A The three-generation pedigree is used to map out risks for genetic diseases in families, as well as Conditions with modifiable risk factors. The review of systems is used to evaluate the history of the Child’s body systems. The genogram is an approach to developing a family database to provide a graphic Representation of family structure, roles, and problems of recurring significance in a family. The coma Is used to identify relationships in the family and community that are supportive or harmful. 3. The primary care pediatric nurse practitioner is performing a well-child check-up on a 20- month-old Child. The child was 4 weeks premature and, according to a parent-completed developmental Questionnaire, has achieved milestones for a 15-month-old infant. Which action is correct? a. Perform an in-depth developmental assessment screen at this visit to evaluate this child. b. Reassure the parent that the child will catch up to normal development by age 2 years. c. Re-evaluate this child's development and milestone achievements at the 2-year visit. d. Refer the child to a specialty clinic for evaluation and treatment of developmental delay. - Correct answer ANS: A This child should be at a 19-month adjusted age for prematurity so, according to the parent screen, is 4 Months behind. The PNP should perform a more in-depth screen to evaluate this delay. Waiting to see if The child will "catch up" or assuring the parent that this will happen will cause the delays to become more Severe. A referral to a specialty clinic should not be made solely on the basis of the parent- completed Questionnaire but only after further evaluation of possible delays. 4. When formulating developmental diagnoses for pediatric patients, the primary care pediatric nurse P a g e 2 | 134 Unless an actual speech delay is observed, a referral is not indicated, nor is it necessary to implement a Home therapy. 12. The primary care pediatric nurse practitioner is performing a well-child assessment on an adolescent And is concerned about possible alcohol and tobacco use. Which assessment tool will the nurse Practitioner use? a. CRAFFT b. HEEADSSS c. PHQ-2 d. RAAPS - Correct answer ANS: A The CRAFFT tool is a six-question tool used to screen for adolescent substance abuse. The HEEADSSS Is used as a psychosocial screening tool. The PHQ-2 is a rapid screen for depression. The RAAPS is used To assess risk behaviors that contribute to most morbidity, mortality, and social problems in teens. 13. The primary care pediatric nurse practitioner is assessing a toddler whose weight and body mass index (BMI) are below the 3rd percentile for age. The nurse practitioner learns that the child does not have Regular mealtimes and is allowed to carry a bottle of juice around at all times. The nurse practitioner Plans to work with this family to develop improved meal patterns. Which diagnosis will the nurse Practitioner use for this problem? a. Failure to thrive b. Home care resources inadequate c. Nutrition alteration - less than required d. Parenting alteration - Correct answer ANS: D Because the PNP is planning to intervene by helping the parents to provide appropriate food habits, the Correct diagnosis should be "Parenting alteration." "Failure to thrive" is a medical diagnosis and requires A medical and social evaluation to rule out organic causes or detect neglect. "Home care resources Inadequate" would be used if the PNP suspects that the family lacks adequate funds to purchase food. "Nutrition alteration" is a NANDA diagnosis and would be used if the PNP planned to consult with a Dietician or give nutritional information. 14. The primary care pediatric nurse practitioner is obtaining a medical history about a child. To integrate Both nursing and medical aspects of primary care, which will be included in the medical history? a. Complementary medications, alternative health practices, and chief complaint b. Developmental delays, nutritional status, and linear growth patterns c. Medication currently taking, allergy information, and family medical history d. Speech and language development, beliefs about health, and previous illnesses - Correct answer ANS: D An assessment model that integrates the nursing and medical aspects of primary care uses three domains: Developmental problems (speech and language development), functional health problems (beliefs about Health), and diseases (chief complaint). The other examples all use domains associated with the traditional Medical model and do not contain nursing aspects associated with functional health problems. 1. The primary care pediatric nurse practitioner learns that an African-American family lives in a Neighborhood with a high crime rate and suggests that they try moving to another neighborhood for The safety of their children. This is an example of A. cultural sensitivity. B. group bias. C. individual privilege. D. racial awareness. - Correct answer ANS: C P a g e 5 | 134 Privilege can be individual- or group-based and refers to the often unconscious lack of understanding of What other groups must deal with. The PNP is not aware that the family may lack the resources to move, May be fearful of moving to a "white" neighborhood, or may even feel safe around people that they know. Cultural sensitivity is an awareness of and respect for other cultures. Group bias is a prejudice, based on Cultural, racial, or ethnic differences, toward a group of people. Racial awareness would describe an Awareness of cultural differences based on race. 2. The primary care pediatric nurse practitioner is examining a child whose parents recently emigrated From a war-torn country in the Middle East. Which is a priority assessment when performing the Patient history? a. Asking about physical, psychological, and emotional trauma b. Determining the parents' English language competency and literacy level c. Learning about cultural preferences and complementary medicine practices d. Reviewing the child's previous health and illness records - Correct answer ANS: A Recent history that includes trauma, loss, and refugee camp experience may exacerbate difficulties Adjusting to life in the U.S. and can lead to acute and chronic physical and mental health concerns. All of The other parts of the history will be necessary, but this should be a priority, since the family has escaped a War-torn country. 3. The primary care pediatric nurse practitioner in a community health center meets a family who has Recently immigrated to the United States who speak only Akron. They arrive in the clinic with a Church sponsor, who translates for them. The pediatric nurse practitioner notices that the sponsor Answers for the family without giving them time to speak. The pediatric nurse practitioner will: A. ask the sponsor to allow the family to respond. B. develop the plan of care and ask the sponsor to make sure it is followed. C. request that the sponsor translate written instructions for the family. D. use the telephone interpreter service to communicate with the family. - Correct answer ANS: D Federally funded managed care networks and community health centers are required to have interpreters Accessible for clients with limited English proficiency. A commercial telephone interpreter service has Been shown to be as effective as an "in-person" interpreter. Relying on family members or community Members may not be reliable and may jeopardize patient confidentiality. This interpreter is answering for The clients without hearing what they have to say, which can compromise care. 4. The primary care pediatric nurse practitioner provides well child care for a community of immigrant Children from Central America. The pediatric nurse practitioner is surprised to learn that some of the Families are Jewish and not Catholic. This response is an example of cultural: A. collectivism. B. constructivism. C. essentialism. D. individualism. - Correct answer ANS: C An essentialist view of culture, which dominates the health care literature, portrays an ethnic minority Group as having a static set of traits and oversimplifies cultural information, applying traits to all members Of the group. Assuming that all people from Central America are Catholic is an example of this Oversimplification. Collectivism refers to a member of an ethnic group who perceives himself or herself To be intrinsically part of that group. A constructive view recognizes culture as complex and dynamic and P a g e 6 | 134 Sees people as individuals who may belong to multiple cultures simultaneously. Individualism recognizes The individual, and not the group, as the basic unit of survival. 5. The primary care pediatric nurse practitioner cares for children from a Native American family and Learns that they used many herbs to treat and prevent illness. Which approach will the pediatric nurse Practitioner use to promote optimum health in the children? a. Ask about the types of practices used and when they are applied. b. Provide a list of harmful herbs and ask the family to avoid those. c. Suggest that the family avoid using these remedies in their children. d. Tell the parents to use the herbs in conjunction with modern medications. - Correct answer ANS: A The challenge, when working with families from different cultural backgrounds, who use alternative or Complementary medicines, is to find ways to achieve a mutual understanding of the differences and to Negotiate an acceptable plan of care. The first step is to begin a discussion about these practices. Providing a list of harmful herbs, suggesting that the family avoid certain herbs, and suggesting that the Herbs are only an adjunct to "modern medicine" will sound disparaging and will convey a sense of Mistrust. 6. A primary care pediatric nurse practitioner working in a community health center wishes to develop a Program to assist impoverished children and families to have access to healthy foods. Which strategy Will the pediatric nurse practitioner employ to ensure the success of such a program? a. Asking community members to assist in researching and implementing a program b. Designing a community garden approach that involves children and their parents c. Gaining support from the corporate community to provide needed resources d. Providing evidence-based information about the importance of a healthy diet - Correct answer ANS: A Community collaboration can be fostered through community-based participatory research (CBPR), Which is transformative research that bridges the gap between science and practice by actively engaging Communities with formally trained researchers. In this type of research, community members formally Participate in all aspects of the process, making the findings more relevant to the community it affects the Most. Designing a community garden approach without first knowing whether the community needs or Wants it does not ensure success. Gaining support from the corporate community without input from the Affected community does not guarantee success. Giving evidence-based information does not involve the Community members in research and does not increase success. 7. The primary care pediatric nurse practitioner works with families from a variety of cultures and Socioeconomic classes. Which is an example of cultural humility in practice? a. Giving health care advice that takes cultural differences into account b. Identification of other cultures that may be superior to one's own culture c. Receptivity to learning about the perspectives of other cultures d. Respecting other cultures while maintaining the views of one's own - Correct answer ANS: C Cultural humility is defined as the lifelong commitment to developing mutually beneficial, No paternalistic partnerships and is based on a model of passive volition, receptivity, and being open to Learning from others. Practitioners who have cultural humility are always seeking to learn about other Cultures. Cultural humility involves asking questions, rather than giving answers. Cultural humility does Not mean identifying one's own culture as inferior. Practitioners who are culturally competent are open to P a g e 7 | 134 b. Perform a developmental assessment and tell the parent which delays are evident. c. Point out the tasks that the infant can perform while conducting the assessment. d. Refer the infant to a developmental specialist for a complete evaluation. - Correct answer ANS: C When discussing developmental delays with parents, it is important to be positive and to initially focus on Strengths. Explaining that developmental delays develop over time is true but does not reassure the parent Or help the parent cope with feelings. Referrals are not indicated unless delays are present and may take Time. 5. Which recommendation will a primary care pediatric nurse practitioner make when parents ask about? Ways to discipline their 3-year-old child who draws on the walls with crayons? a. Give the child washable markers so the drawings can be removed easily. b. Provide a roll of paper for drawing and teach the child to use this. c. Put the child in "timeout" each time the child draws on the walls. d. Take the crayons away from the child to prevent the behavior. - Correct answer ANS: B Discipline involves training or education that molds appropriate behavior and is used to teach the child What is permitted and encouraged. Providing an appropriate outlet for drawing helps to teach the child Where to use the crayons. Using washable markers allows the parents to clean the walls but does not teach The child appropriate behaviors. Timeout and taking away the crayons are forms of punishment, or a loss Of privileges, that are administered as a form of retribution. 6. The primary care pediatric nurse practitioner enters an exam room and finds a 2-month-old infant in a Car seat on the exam table. The infant's mother is playing a game on her smart phone. The nurse Practitioner interprets this behavior as: A. a sign that the mother has postpartum depression. B. extremely concerning for potential parental neglect. C. of moderate concern for parenting problems. D. within the normal range of behavior in early parenthood. - Correct answer ANS: C A parent who seems disinterested in a child raises moderate concerns for parenting problems. It does not Necessarily signal postpartum depression. It is not a mark for extreme concern. It is not within the Expected range of behaviors. 7. During a well-child assessment of an 18-month-old child, the primary care pediatric nurse practitioner Observes the child becoming irritable and uncooperative. The parent tells the child to stop fussing. What will the nurse practitioner do? a. Allow the parent to put the child in a "timeout." b. Ask the parent about usual discipline practices. c. Offer the child a book or a toy to look at. d. Stop the exam since the child has reached a "meltdown." - Correct answer ANS: C The child has exhibited early signs of misbehavior. At this stage, distraction and active engagement may Be used to stop more problems from occurring. It is not necessary to use a timeout because the child Hasn’t reached the point where cooperation is impossible. The PNP should model appropriate Interventions by offering the child a distraction and may ask the parent about discipline practices later in The visit. The child is not at a "meltdown" state. 8. The primary care pediatric nurse practitioner performs a physical examination on a 9-month- old infant And notes two central incisors on the lower gums. The parent states that the infant nurses, takes solid Foods three times daily, and occasionally takes water from a cup. What will the pediatric nurse? P a g e 10 | 134 Practitioner counsel the parent to promote optimum dental health? a. To begin brushing the infant's teeth with toothpaste b. To consider weaning the infant from breastfeeding c. To discontinue giving fluoride supplements d. To make an appointment for an initial dental examination - Correct answer ANS: D The American Academy of Pediatric Dentistry recommends a first dental examination at the time of Eruption of the first tooth and no later than 12 months old. Parents should be counseled to clean the Infant’s teeth but with water only. Weaning from breastfeeding is not indicated, although mothers should Not let the infant nurse while sleeping to prevent milk from bathing the teeth. Fluoride supplements Should not be discontinued. 9. The primary care pediatric nurse practitioner has a cohort of patients who have special health care Needs. Which is an important role of the nurse practitioner when caring for these children? a. Care coordination and collaboration b. Developing protocols for parents to follow c. Monitoring individual education plans (IEPs) d. Providing lists of resources for families - Correct answer ANS: A Care coordination is one of the key elements for children with special health care needs. PNPs are Especially suited for this role and have the unique skills to function as care coordinators. Care for these Children should involve shared decision making and individualized care and not "cookbook" approaches. The PNP may advocate for children's health care needs for the IEP but does not monitor these. The PNP Should not just give parents lists of phone numbers but should assist them to make appointments. 10. A single mother of an infant worries that living in a household with only one parent will cause her Child to be maladjusted. To help address the mother's concerns, the primary care pediatric nurse Practitioner will suggest: A. developing consistent daily routines for the child. B. exposing her child to extended family members when possible. C. not working outside the home during the first few years. D. taking her child to regular play date activities with other children. - Correct answer ANS: A Providers can teach parents that providing predictable, consistent, and loving care helps an infant to learn Trust and help influence positive brain development. Involving extended family members and going to Play dates are good ways to socialize children but are not essential to learning trust. It may not be possible For her to be a stay-at-home mother. 1. The primary care pediatric nurse practitioner performs a well-baby examination on a 7-day- old infant Who is nursing well, according to the mother? The nurse practitioner notes that the infant weighed 3250 grams at birth and 2990 grams when discharged on the second day of life. The infant weighs 3080 grams at this visit. Which action is correct? a. Follow up at the 2-month checkup. b. Refer to a lactation consultant. c. Schedule a weight check in 1 week. d. Suggest supplementing with formula. - Correct answer ANS: C This infant lost about 8% of its birth weight, which is normal and, since discharge home, has gained at Least 15 grams per day, which is also normal. The PNP should schedule a weight check in a week to make Sure the infant regains its birth weight, since most should regain this in 10 to 14 days and since this loss of Birth weight is at the high end of normal. It is not necessary to refer to a lactation consultant or P a g e 11 | 134 Supplement with formula, since the infant is gaining weight adequately. 2. The parent of a newborn infant asks the primary care pediatric nurse practitioner when to intervene to Help the infant's future intellectual growth. What will the nurse practitioner tell the parent? a. Cognitive learning begins during the toddler years. b. Intellectual growth begin when speech develops. c. Language and literacy skills begin at birth. d. Preschool is an optimal time to begin general learning. - Correct answer ANS: C General learning and acquisition of skills for later reading and writing begin at birth, not in kindergarten Or first grade, and these skills grow with everyday loving interactions between infants and caregivers. Cognitive learning changes during toddler years but begins at birth. Intellectual growth is not tied to Speech alone. 3. During an assessment of a 4-week-old infant, the primary care pediatric nurse practitioner learns that a Breastfed infant nurses every 2 hours during the day but is able to sleep for a 4-hour period during the Night. The infant has gained 20 grams per day in the interval since last seen in the clinic. What will the Nurse practitioner recommend? a. Continuing to nurse the infant using the current pattern b. Nursing the infant for longer periods every 4 hours c. Supplementing with formula at the last nighttime feeding d. Waking the infant every 2 hours to nurse during the night - Correct answer ANS: A Infants who are encouraged to breastfeed every 2 to 3 hours may have one longer stretch of 4 hours at Night. This infant is gaining between 0.5 and 1 gram per day, which is appropriate. It is not necessary to Alter the pattern of nursing or to supplement with formula. 4. The mother of a 6-week-old breastfeeding infant tells the primary care pediatric nurse practitioner that Her baby, who previously had bowel movements with each feeding, now has a bowel movement once Every third day. What will the nurse practitioner tell her? a. Her baby is probably constipated. b. It may be related to her dietary intake. c. She should consume more water. d. This may be normal for breastfed babies. - Correct answer ANS: D Infants begin to have fewer bowel movements and may have bowel movements ranging from once or Twice daily to once every other day when breastfed. Unless there are other signs, the baby is probably not Constipated. The mother does not need to change her intake of foods or water, unless constipation is Present. 5. The mother of a 3-month-old child tells the primary care pediatric nurse practitioner that it is "so Much fun" now that her infant coos and smiles and wants to play. What is important for the nurse? Practitioner to teach this mother? a. Appropriate ways to stimulate and entertain the infant b. How to read the infant's cues for overstimulation c. The importance of scheduling "play dates" with other infants d. To provide musical toys to engage the infant - Correct answer ANS: B By 3 months, infants demonstrate a social smile and will become more active, alert, and responsive. Parents may mistakenly assume that the infant can handle more activity and stimulation when this occurs, And the PNP should teach caregivers how to recognize infant cues for the need to rest or to have Decreased stimulation. P a g e 12 | 134 Switch" at times for clarity as they speak. They seem to understand that not everyone has this Ability. Most children who are bilingual develop a dominant language. 1. 4. The parents of a 3-year-old child are concerned that the child has begun refusing Usual foods and wants to eat mashed potatoes and chicken strips at every meal and snack. The child's rate of weight has slowed, but the child remains at the same percentile for weight On a growth chart. What will the primary care pediatric nurse practitioner tell the parents to? Do? a. a. Allow the child to choose foods for meals to improve caloric intake. b. b. Place a variety of nutritious foods on the child's plate at each meal. c. c. Prepare mashed potatoes and chicken strips for the child at mealtimes. d. d. Suggest cutting out snacks to improve the child's appetite at mealtimes. - Correct answer ANS: B Young children should have three meals and two nutritious snacks each day. The parents' Responsibility is to provide nutritious foods and allow children to choose how much they will eat. Children who are allowed to choose foods will likely make selections that are not healthy. Parents should be discouraged from preparing separate meals for their children. Snacks are Necessary to maintain adequate intake and energy. 1. 5. The parent of a 24-month-old child asks the primary care pediatric nurse Practitioner when toilet training should begin. How will the pediatric nurse practitioner Respond? a. a. "Begin by reading to your child about toileting." b. b. "Most children are capable by age 2 years." c. c. "Tell me about your child's daily habits." d. d. "We should assess your child's motor skills." - Correct answer ANS: C To assess the parent's understanding of toilet readiness, the nurse practitioner will ask the Parents about the child's daily habits and routines to see if the child has predictable patterns That can be the basis for toilet training. While providing storybooks about toileting can help Children learn, the first step is to assess toilet readiness. Even though many children are Capable at this age, evaluating personal readiness is key to beginning toilet training. Assessment of motor skills may be a second step. 1. 6. The primary care pediatric nurse practitioner is counseling the parents of a toddler About appropriate discipline. The parents report that the child is very active and curious, and They are worried about the potential for injury. What will the pediatric nurse practitioner? Recommend? a. a. Allow the child to explore and experiment while providing appropriate limits. b. b. be present while the child plays to continually teach the child what is Appropriate. C. c. Let the child experiment at will and to make mistakes in order to learn. d. d. Say "no" whenever the child does something that is not acceptable. - Correct answer ANS: A The child who is securely attached uses the parents as a base from which to safely explore the World. Toddlers learn by doing and need to experiment to gain mastery over the environment. It Is important that parents are present for safety, but parents should not be ever-present and Controlling. Parents should be close by and should intervene if the child is at risk for injury. Continual criticism and the use of the word "no" can make the toddler feel powerless. 1. 7. The primary care pediatric nurse practitioner performs a physical examination on A 9-month-old infant and notes two central incisors on the lower gums. The parent states That the infant nurses, takes solid foods three times daily, and occasionally takes water from A cup. What will the pediatric nurse practitioner counsel the parent to promote optimum? Dental health? A. a. To begin brushing the infant's teeth with toothpaste B. b. To consider weaning the infant from breastfeeding C. c. To discontinue giving fluoride supplements D. d. To make an appointment for an initial dental examination - Correct answer ANS: D The American Academy of Pediatric Dentistry recommends a first dental examination at the Time of eruption of the first tooth and no later than 12 months old. Parents should be counseled To clean the infant's teeth but with water only. Weaning from breastfeeding is not indicated, Although mothers should not let the infant nurse while sleeping to prevent milk from bathing the Teeth. Fluoride supplements should not be discontinued. 1. 8. During a well-child assessment of an 18-month-old child, the primary care Pediatric nurse practitioner observes the child point to a picture of a dog and say, "Want Puppy!" The nurse practitioner recognizes this as an example of a. a. holophrastic speech. P a g e 15 | 134 b. b. receptive speech. c. c. semantic speech. d. d. telegraphic speech. - Correct answer ANS: D Syntax, or the structure of words in sentences or phrases, is developed in stages between the Ages of 8 months and 3.5 years. Telegraphic speech begins at about 18 months of age when Children speak in phrases with many words omitted, so that the sentence sounds like a Telegram. Holophrastic speech is the use of a single word to express a complete idea. Receptive speech refers to the ability to understand a word without necessarily being able to Use the word. Semantics is the understanding that words have specific meanings. 1. 9. The primary care pediatric nurse practitioner is evaluating a 2-year-old with a Documented speech delay. Screenings to assess motor skills and cognition are normal, and The child passed a recent hearing test. What will the pediatric nurse practitioner do next? a. a. Ask the child's parents whether they read to the child. b. b. Give parents educational materials to encourage speech. c. c. Refer the child to an early intervention program. d. d. Suggest that they purchase age-appropriate music videos. - Correct answer ANS: A Language development requires oral-motor ability, auditory perception, and cognitive ability, Which this child has been shown to have, as well as the psychosocial-cultural environment to Motivate the child to engage in language use. The PCPNP's initial step should be to determine Whether the parents provide such an environment. Educational materials may be used after it is Determined that these are useful. Early intervention may be used if the speech delay persists. Music videos do not necessarily engage the child in expression of speech. 1. 10. The mother of a 3-year-old child takes the child to a play group once a week. She expresses concern that the child plays with toys but does not interact with the other Toddlers. What will the primary care pediatric nurse practitioner counsel the mother? A. a. The child probably is very shy but will outgrow this tendency with repeated Exposure to other children. B. b. The toddler may have a language delay that interferes with socialization with Other children. c. c. Toddlers may be interested in other children but usually do not engage in Interactive play. d. d. Toddlers need more structured play to encourage interaction and socialization With others. - Correct answer ANS: C Parallel play is common among toddlers who, although they may be fascinated by other Children, generally do not engage with peers in an interactive manner. This does not mean that The child is shy or has a language delay, although in preschool years, the development of Symbolic language increases interactive play. Children need both structured and free play, but Structured play will not increase interaction during this normally parallel period. 1. The primary care pediatric nurse practitioner is preparing to conduct a well-child assessment of an 8- Year-old child. How will the nurse practitioner begin the exam? a. Ask the child about school, friends, home activities, and sports b. Discuss the purpose of the visit and explain the procedures that will be performed c. Offer age-appropriate information about usual developmental tasks d. Provide information about healthy nutrition and physical activities - Correct answer ANS: A To build rapport with the child and parent, the PNP will begin by asking direct questions to the child, Encouraging the child to share information about daily routines. The other answers list aspects of the well Child visit that can be introduced after the initial conversation. 2. The primary care pediatric nurse practitioner is examining a 6-year-old child who attends first grade. The child reports "hating" school. The parent states that the child pretends to be sick frequently in Order to stay home from school. To further assess this situation, the nurse practitioner will first ask the Child: A. about school performance and grades. B. why school is so distressing. C. to name one or two friends. D. whether bullying is taking place. - Correct answer ANS: C The earliest school-age psychosocial milestone occurs when children learn to separate easily from family, P a g e 16 | 134 Allowing them to go to school. Mastery of these skills enables them to develop and maintain peer Friendships. Social interaction skills are necessary in order to develop mastery over school activities. Asking the child to describe why school is distressing may not elicit information, since the child may not Be able to articulate this. Bullying is not the only reason for disliking school, but, if it is, will emerge During a discussion about friends and schoolmates. 3. A school-age child has begun refusing all cooked vegetables. What will the primary care pediatric? Nurse practitioner recommend to the parent? a. Allow the child to make food choices since this is usually a phase b. Ensure that the child has three nutritious meals and two nutritious snacks each day c. Prepare vegetables separately for the child to encourage adequate intake d. Teach the child how important it is to eat healthy fruits and vegetables - Correct answer ANS: B Children have food jags that are generally self-limited. The parent's responsibility is to provide three Nutritious meals and two nutritious snacks each day so that all available choices are acceptable. Allowing Food choices may result in an overabundance of non-nutritious foods selected. It is not necessary to Prepare separate dishes for a child who is going through a temporary phase. Teaching the child about Nutrition is important but will not likely have much impact during this phase. 4. The parent of a 6-year-old child expresses concern that the child may have ADHD. Which screening Tool will the primary care pediatric nurse practitioner use to evaluate this possibility? a. Behavioral and Emotional Screening System for Children (BESS-2) b. Behavioral Assessment for Children - 2nd ed. (BASC-2) c. Conner's 3 Parent and Teacher Rating Scale D. Pediatric Symptom Checklist (PSC) - Correct answer ANS: C The Conner's Parent and Teacher Rating Scale is used to assess ADHD symptoms in children aged 6 to 18 years. The BESS-2 is used to evaluate social emotional and mental health in children. The BASC-2 is Used to further assess children who have positive findings on the BESS-2. The PSC is used to assess Cognitive, emotional, and behavioral problems in children. 5. The primary care pediatric nurse practitioner performs a physical examination on a 12-year- old child And notes poor hygiene and inappropriate clothes for the weather. The child's mother appears clean And well dressed. The child reports getting 6 to 7 hours of sleep each night because of texting with Friends late each evening. What action by the nurse practitioner will help promote healthy practices? a. Discuss setting clear expectations about self-care with the mother b. Give the child information about sleep and self-care c. Reassure the mother that this "non-compliance" is temporary d. Tell the mother that experimenting with self-care behaviors is normal - Correct answer ANS: A Parents of school-age children should be advised to set clear limits for their children for cleanliness, Healthy exercise, hours of sleep, and other health promotion behaviors to encourage the development of Responsibility for these things. Giving the child information can be done along with setting expectations, But, at this age, the parent should still be supervising. While "non-compliance" is a part of this process, And is a means of asserting independence, parents need to discuss this with children to resolve the issue. P a g e 17 | 134 Be useful as well, but routines and special activities are most important. 1. The primary care pediatric nurse practitioner is examining a 15-year-old female who reports having Her first period at age 13. She states that she has had five periods in the last year, with the last one 2 Months prior. She participates in basketball at school. Which action is correct? A. Perform biometric screening to determine lean body mass. b. Prescribe oral contraceptives pills to regulate her periods. c. Reassure her that this is perfectly normal at her age. d. Refer her to an endocrinologist for hormonal evaluation. - Correct answer ANS: A Although it can take 18 to 24 months for adolescents to establish regulatory cycles, periods can also be Affected by athletic activity that decreases body fat. The PNP should assess the percentage of lean body Mass, which should be 75% or less to maintain regular ovulatory cycles. OCPs are useful for regulating Periods if this persists and other causes are ruled out. It is not necessary to refer her to an endocrinologist Unless problems persist in spite of standard management. 2. The parent of a 14-year-old child tells the primary care pediatric nurse practitioner that the child skips Classes frequently in spite of various disciplinary measures, such as grounding and extra homework And is earning Cs and Ds in most classes. What will the nurse practitioner recommend? a. Counseling for emotional problems b. Development of an Individual Education Plan c. Evaluation for possible learning disorders d. Referral for a behavioral disorder - Correct answer ANS: C Frequent school absenteeism, class skipping, and other types of school avoidance may indicate a problem With cognitive ability and should be assessed. When cognitive disorders are ruled out, other issues, such As behavioral and emotional problems may be considered. IEPs are used for children who have identified Special physiological or cognitive needs and may be useful if a cognitive disorder is identified. 3. The primary care pediatric nurse practitioner is performing a well-child exam on a 12-year-old female Who has achieved early sexual maturation? The mother reports that she spends more time with her Older sister's friends instead of her own classmates. What will the nurse practitioner tell this parent? a. Early-maturing girls need to identify with older adolescents to feel a sense of belonging. b. Girls who join an older group of peers may become sexually active at an earlier age. c. Spending time with older adolescents indicates a healthy adjustment to her maturing body. d. The association with older adolescents will help her daughter to gain social maturity. - Correct answer ANS: B While it is true that early maturing females may join an older group of peers to feel that they fit in, the Ones who do put themselves at risk for risky behaviors, including sexual activity. Although many teens Feel awkward when they mature at different rates than their peers, joining a group of older peers Demonstrates a poor adjustment and does not promote social maturity. 4. The primary care pediatric nurse practitioner is performing a well-child assessment on a 13- year-old Female whose mother asks when her daughter's periods may start? Which information will the nurse Practitioner use to help estimate the onset of periods? a. The age of the mother's menarche b. The patient's age at thelarche c. When adrenarche occurred d. Whether linear growth has stopped - Correct answer ANS: B Thelarche, or the development of breast budding, generally precedes menarche by 2.5 years, so this P a g e 20 | 134 Should be determined when attempting to predict this milestone. The age of the mother at menarche is not A reliable indicator. Adrenarche is related to adrenal and not gonadal development and is less valid than Other secondary sex characteristics in assessing sexual maturation. Rapid linear growth usually begins After thelarche and peaks about a year later but is not used to predict menarche. 5. The mother of a 15-year-old adolescent female tells the primary care pediatric nurse practitioner that Her daughter has extreme mood swings prior to her periods, which the adolescent vehemently denies. When asked if she notices anything different just before her periods, the adolescent points to her Mother and says, "She gets really hard to live with." This demonstrates which characteristic of Adolescent thinking? a. Apparent hypocrisy b. Imaginary audience c. Overthinking d. Personal fable - Correct answer ANS: A Apparent hypocrisy is the notion that rules apply differently to adolescents than to others. The adolescent Who chalks up the conflict with her mother related to her premenstrual mood swings does not see her own Role in the conflict. Imaginary audience is the perception that everyone is thinking about them. Personal Fable is the idea that they are special. Overthinking involves making things more complicated than they Need to be. 6. During a well-child assessment of a 13-year-old male, the primary care pediatric nurse practitioner Notes small testicles and pubic and axillary hair. To further evaluate these findings, the nurse Practitioner will ask the patient about A. alcohol and tobacco use. B. changes in voice. C. increase in height and weight. D. participation in sports. - Correct answer ANS: D The initial sign of puberty in males is testicular enlargement. If this does not precede other changes, the PNP should consider whether the boy is taking exogenous anabolic steroids, common among those who Wish to improve athletic ability. These findings are not concerning for alcohol or tobacco use. Voice Changes and rapid growth may occur with pubic hair development, but the primary concern is anabolic Steroid use. 7. The parent of an adolescent reports noting cutting marks on the teen's arms and asks the primary care Pediatric nurse practitioner what it means. What will the nurse practitioner tell this parent? A. Cutting is a way of dealing with emotional distress. b. It is a method of fitting in with other adolescents. c. The behavior is common and will usually stop. d. This type of behavior is a type of suicide attempt. - Correct answer ANS: A Self-injurious behavior (SIB) is used as a coping strategy to relieve distress, anger, and stress. It is not Commonly done among adolescents and is not a way of fitting in with a peer group. Because it indicates Underlying distress, adolescents must get help identifying these causes. Many have a history of physical, Sexual, or emotional abuse. Although individuals who engage in SIB are more likely to attempt suicide in The future, the act itself is not a suicide attempt. 8. The parent of a 14-year-old child tells the primary care pediatric nurse practitioner that the adolescent P a g e 21 | 134 Has expressed a desire to be a vegetarian, is refusing all meat served at home, and wants the family to Eat vegetarian meals. What will the nurse practitioner tell the parent? a. Do not allow a vegetarian diet in order to maintain appropriate limits for the adolescent. b. Provide vegetarian options for the adolescent that preserve adequate nutrition and protein intake. c. Suggest that the adolescent prepare appropriate vegetarian dishes to complement family meals. d. Tell the adolescent that a vegetarian diet may be considered in adulthood but not while living at Home. - Correct answer ANS: C Early adolescents begin to develop their own value system and may try value systems other than the one That they have learned from their family, which is a normal part of establishing personal identity. The Parent may allow expression of other values, such as a vegetarian diet, as long as nutritional needs are met And the adolescent takes responsibility for preparing the food. 9. The primary care pediatric nurse practitioner is performing an exam on an adolescent male who asks About sexual identity because of concern that a friend is worried about being gay. Which response will? The nurse practitioner make in this situation? a. Provide the teen with a questionnaire to gain information about his sexuality. b. Remind the adolescent that mandatory reporting requires disclosure to parents. c. Suggest that the adolescent discuss sexual concerns with his parents. d. Tell the adolescent that, unless he is at risk, what he says will be confidential. - Correct answer ANS: D Adolescents should be encouraged to divulge information about their sexuality to providers by assuring Them that confidentiality will be maintained unless the health of the child or others is at risk. The Adolescent may be trying to ask questions about himself in a manner that doesn't implicate his own Sexuality, so the PNP should attempt to gain his confidence. Questionnaires may be useful when Collecting information, but this adolescent has already begun a discussion about the topic. An adolescent Who is concerned about being gay may not be ready to come out to his parents. 10. The primary care pediatric nurse practitioner is performing a well-child exam on a 17-year- old female Whose mother is present during the history? The mother expresses concern that her daughter wishes to Have an eyebrow piercing and states that she is opposed to the idea. What will the nurse practitioner? Do? a. Provide information about piercings and encourage continued discussion. b. Remind the adolescent that her mother is responsible for her health. c. State that piercings are relatively harmless and are an expression of individuality. d. Suggest that she wait until she is 18 years old and can make her own decisions. - Correct answer ANS: A Adolescents who pierce their noses or have strange haircuts may be irritating to parents, but these are Ways of expressing individuality and help them to achieve psychosocial milestones. The fact that the teen And her mother are discussing this is a good sign that the adolescent isn't in complete rebellion. The PNP Should provide accurate health information and encourage continued dialogue. Although it is true that Piercings are relatively harmless, the PNP shouldn't "side" with the teen during an open discussion or tell The teen that the mother is "in charge." 11. The mother of a 16-year-old male was recently divorced after several years of an abusive relationship And tells the primary care pediatric nurse practitioner that the adolescent has begun skipping school P a g e 22 | 134 Tells the primary care pediatric nurse practitioner that he doesn't want to give up sweets and soft Drinks because he enjoys them too much. Which stage of change does this represent? a. Action b. Contemplation c. Precontemplation d. Preparation - Correct answer ANS: B The contemplation stage occurs when the individual is aware that a problem exists and struggles with the Costs and energy required for change. This client knows what to do but doesn't want to give up the Enjoyment of certain treats. The action phase occurs when behaviors to eliminate the problem occur. The Precontemplation stage occurs when the individual wishes to change but is resistant to change without Clear reason for being so. The preparation stage occurs when the individual makes small behavior Changes, such as giving up sweetened soda first, in preparation for commitment to the actual plan. 7. The primary care pediatric nurse practitioner provides patient teaching for children newly diagnosed With irritable bowel syndrome (IBS). At which stage of development will children be able to? Understand the link between stress and the symptoms of the disease? a. Concrete-operational stage b. Formal-operational stage c. Pre-conceptual stage d. Sensorimotor stage - Correct answer ANS: B In the formal-operational stage, children can understand how the body works and how illness may be Related to the body and the environment. It is late in this stage when children can conceptualize the mind And body interactions of illness. In the concrete-operational stage, children do not yet distinguish between The body and the mind. The pre-conceptual stage is characterized by a perception that people must be near The cause of an illness. The sensorimotor stage, causes are not considered. 8. The primary care pediatric nurse practitioner is counseling an obese 16-year-old client about weight Management. The adolescent says, "I know I need to lose weight, but I don't want to give up all my Favorite foods." When using motivational interviewing techniques, how will the nurse practitioner Respond? a. "Do you think there are any foods you could limit or do without for a while?" b. "I hear you telling me that you really don't have a desire to lose weight." c. "If you can't give up these foods, you won't see the benefits of weight loss." d. "In the long run, the sacrifices you make today will improve your health." - Correct answer ANS: A Motivational interviewing (MI) uses persuasion, rather than coercion, drawing out the individual's Motivation for change in order to build a collaborative partnership between the client and the counselor. This adolescent has indicated an understanding of the need to lose weight but doesn't want to give up Favorite foods. The PNP asks if there are any foods, allowing the client to choose possible options. In Answer B, the PNP does not acknowledge the client's motivations. In the other two answers, the PNP Doesn’t allow choices. 9. The primary care pediatric nurse practitioner is working with a 12-year-old female who has poor Diabetes control. The child tells the nurse practitioner that the parent forgets to remind her to check Her blood sugars. Which action is correct? a. Assess the parent's knowledge about diabetes management. b. Help the child develop a strategy to remember without parental prompts. c. Refer to a social worker to help the family overcome obstacles to care. P a g e 25 | 134 d. Remind the child's parent about the importance of good diabetes control. - Correct answer ANS: B When adherence is a concern, the PNP should work with the family or the child to develop intervention Strategies appropriate to the skills, needs, and desires of the parents and children. A 12-year- old child is Old enough to take responsibility for checking blood glucose, so the PNP can work with her. The other Options may be used if the situation does not improve. 10. The primary care pediatric nurse practitioner is performing a focused problem assessment on a child Who has asthma and learns that one of the child's parents smokes around the child in spite of being Advised against this. The nurse practitioner recognizes this as a possible alteration in which functional Health pattern? a. Cognitive-perceptual b. Health perception c. Role-relationship d. Values-beliefs - Correct answer ANS: B The health perception-health management pattern describes client perceptions of personal health and Health behaviors and includes the belief that there is a relationship between health status and health Practices. The cognitive-perceptual pattern describes sensory-perceptual and cognition patterns. The role- Relationship pattern describes patterns of roles and responsibilities of the client and other family members. The values-beliefs pattern identifies the beliefs that influence daily living, decision making, and meaning Of life. 11. The parent of a newborn has quit smoking cigarettes within the past month and reports feeling fidgety. Using a "reframing" technique, how will the primary care pediatric nurse practitioner respond? a. Explore ways that the parent can use this extra energy to do things for the baby. b. Remind the parent that this is a normal, temporary part of nicotine withdrawal. c. Suggest that the parent take up exercise to enjoy the benefits of not smoking. d. Tell the parent that, over time, these symptoms of withdrawal will subside. - Correct answer ANS: A Reframing is a counseling strategy that uses the context of an experience to give it a new meaning, Creating a frame of reference that focuses on a desired outcome instead of the current problem. The Withdrawal symptoms associated with tobacco cessation are uncomfortable, but the PNP can suggest Channeling this nervous energy into positive action for the baby. Telling the parent that the symptoms of Withdrawal are temporary or normal does not reframe the perception. Suggesting exercise may be Beneficial, but does not reframe the situation. 12. The primary care pediatric nurse practitioner is counseling a school-age child about asthma Management strategies. The child states that it is "too much trouble" to remember to use an inhaled Corticosteroid medication twice daily and reports feeling fine, in spite of exhibiting expiratory Wheezes. Which action uses the health belief and self-efficacy model to teach this child about asthma? Management? a. Asking the child to try to use the inhaler at least once daily b. Discussing whether the child wants to participate in athletics c. Obtaining pre- and post-treatment spirometer testing D. Providing written information about inhaled corticosteroids - Correct answer ANS: C In the health belief model, clients need to believe that taking some action will reduce the risk of P a g e 26 | 134 Symptoms and that the benefits of the action will outweigh the costs or effort. Demonstrating pre- and Post-treatment spirometer measures can help the child see that symptoms are reversible. Asking the child To make one change may be part of the Trans theoretical model, in which small behavior changes may Precede commitment to the actual plan. Discussing athletics may be useful when using the health Promotion model to encourage the client to participate in behaviors that will promote healthy activities, Such as sports. Providing written information is not part of any of the health behavior models. 1. 1. The primary care pediatric nurse practitioner is examining a newborn who is Breastfeeding and notes the presence of an ankyloglossia. What will the nurse practitioner? Do next? a. a. Ask the mother if the infant has any feeding difficulties. b. b. Refer the infant for a possible frenulectomy. c. c. Schedule an appointment with a lactation consultant. d. d. Suggest that the mother feed breast milk by bottle. - Correct answer ANS: A Infants with ankyloglossia may have difficulty feeding if the tongue does not extend well. The PNP should first assess feeding difficulties and then may refer for a lactation consultant or Consider a frenulectomy. 1. 2. The mother of a 2-month-old infant tells the primary care pediatric nurse Practitioner that she is afraid her breast milk is "drying up" because her baby never seems Satisfied and wants to nurse all the time. Which action is correct? a. a. Recommend pumping her breasts after feedings. b. b. Refer the mother to a lactation consultant. c. c. Suggest supplementation with formula. d. d. Weigh the infant to assess for a growth spurt. - Correct answer ANS: D Infants have growth spurts about every 3 to 4 weeks that increase their breast milk needs. Until The mother's milk supply catches up, the infant will act hungry and want to nurse more Frequently. The PNP should evaluate for this growth spurt and then instruct the mother to feed Her baby more often to increase her milk supply. Since the infant is hungry, the infant should Nurse. It is not necessary to refer for a lactation consultation or to supplement with formula. 1. 3. The primary care pediatric nurse practitioner is performing an assessment on a 1- Week-old newborn with a slightly elevated bilirubin who is breastfeeding well and who has Gained 30 grams in the past 24 hours. The infant is stooling and voiding well. The nurse Practitioner suspects breast milk jaundice. Which action is correct? a. a. Order home phototherapy and closely monitor bilirubin levels. b. b. Reassure the mother that the bilirubin level will drop in a few days. c. c. Recheck the serum bilirubin and infant's weight in 24 hours. d. d. Recommend that the mother pump her breast milk for a couple of days. - Correct answer ANS: C Infants with breast milk jaundice who are gaining weight and thriving should continue to Breastfeed and be monitored for the development of pathologic jaundice. It is not necessary to Order phototherapy or discontinue breastfeeding unless pathologic jaundice is present. The Bilirubin may remain elevated up to 3 months. 1. 4. The mother of a newborn asks the primary care pediatric nurse practitioner about The benefits of breastfeeding. What will the nurse practitioner tell her? a. a. Breastfeeding for 9 months or longer will reduce the incidence of food Allergies. b. b. Breast milk is an excellent source of vitamin D, iron, and other essential Nutrients for the baby. c. c. nursing her baby exclusively for at least 4 months will help her infant to resist Infections. D. d. There is a decreased risk of atopic dermatitis in babies who nurse for 12 Months or longer. - Correct answer ANS: C There is evidence that infants who exclusively breastfeed for at least 4 months have less risk for Infection than infant’s breastfed for less time. However, infants who breastfeed exclusively for 9 Months or for longer than 12 months may have increased risks for food hypersensitivities and Atopic dermatitis. Breast milk is a poor source of vitamin D and iron. 1. 5. The primary care pediatric nurse practitioner sees a 3-day-old nursing infant Whose newborn metabolic screen is positive for galactosemia? The nurse practitioner refers The newborn to a specialist for immediate evaluation and will tell the mother P a g e 27 | 134 The emphasis of management of encopresis in the absence of pathology is to establish a regular bowel routine after bowel evacuation. Parents should be taught to use the maintenance laxative for at least 2 months after resolution of the constipation. Referral to a mental health consultant should be reserved for situations where the child and family may have underlying issues contributing to an ongoing problem. It is not necessary to spend time attributing cause or blame unless the problem continues to be chronic following initial management strategies that include maintenance medications. Parents should be taught what the dynamics of encopresis, including retention, are and that they are involuntary. 5. The primary care pediatric nurse practitioner is performing a well child exam on a 12-month- old infant. The parent tells the nurse practitioner that the infant has predictable bowel and bladder habits and asks about toilet training. What will the nurse practitioner tell this parent? a. It is too early to begin introducing the child to the toilet, and the parent should wait until the child is at least 2 years old. b. Placing the child on a "potty" chair helps the child associate elimination cues with the toilet. c. Predictability of elimination patterns indicates readiness for toilet training, and the parent can begin this process. d. The parent should wait until other signs of toilet training readiness occur before introducing the child to the toilet. - Correct answer ANS: B Once elimination patterns are predictable, the parent may place the child on a potty chair to help the child associate elimination with the toilet, as long as the parent understands that this is not actual potty training. It is not too early to begin this association exercise. Predictability of elimination patterns does not indicate cognitive readiness for toilet training. 6. The primary care pediatric nurse practitioner is performing a well child exam on a 24-month- old child. The parent tells the nurse practitioner that the child is being toilet trained and expresses frustration that on some days the child uses the toilet every time and on other days not at all. What will the nurse practitioner do? a. Advise the parent to make the child get clean clothes after an accident. b. Ask the parent about the child's toilet habits and understanding of toilet training. c. Recommend using an awards system to encourage toilet use. d. Suggest that the parent place the child on the toilet at predictable intervals. - Correct answer ANS: B Children often will not toilet train easily if the process is started too early. The PNP should assess toilet training readiness to see if the child is ready. If the child is not ready, toilet training can be very stressful to both the child and the parent(s), so the PNP should not make recommendations that add to this stress. 7. The parent of a 5-year-old child tells the primary care pediatric nurse practitioner that the child has been using the toilet to urinate for since age 3 but continues to defecate in "pull-ups." The nurse practitioner learns that the child has predictable bowel movements and a physical examination is normal. What will the nurse practitioner recommend? a. Providing a reward system to offer incentives when the child uses the toilet b. Put the child back in diapers and resume toilet training in a few months. c. Putting the child on the toilet for 5 to 10 minutes at the usual time of defecation d. Use of polyethylene glycol until the child is able to use the toilet regularly - Correct answer ANS: C Since this child has predictable bowel patterns, the parent can put the child on the toilet for 5 to 10 P a g e 30 | 134 minutes at these times to encourage toilet use. Rewards may be used at some point, but it is not recommended since the child is learning to do what is to be expected. Younger children may be put back in diapers and retrained in a few months. The child is not constipated and does not need medication. 8. The primary care pediatric nurse practitioner evaluates a 4-year-old girl whose parent reports frequent urination in the evenings on weekdays, incontinence after voiding. The parent reports that the child has soft formed stools 5 or 6 times weekly. Which assessment will the nurse practitioner make initially? a. Examination for labial adhesions b. Palpation for abdominal masses c. Screening for potential child abuse d. Urine culture and sensitivity - Correct answer ANS: A The child exhibits incontinence after voiding, or vaginal voiding, which may indicate labial adhesions. Examination for this may be completed easily during the physical assessment. Since the parent reports normal stools, it is less likely that chronic constipation is causing dysfunctional voiding. Screening for child abuse may be necessary if physiologic causes are ruled out. If a UTI is suspected, the first test will be a urinalysis, not a culture. 9. The primary care pediatric nurse practitioner is discussing toileting issues with the parent of a 3-year- old toddler who reports that the child has been toilet trained for several months but has recently been refusing to have bowel movements and is becoming constipated. What will the nurse practitioner do? a. Ask the parent about bathroom facilities in the child's day care. b. Refer the child to a gastroenterologist for evaluation of pathology. c. Suggest putting the child in diapers and resuming toilet training in a few weeks. d. Tell the parent that this represents a developmental delay. - Correct answer ANS: A The child has bowel dysfunction that may be related to restricted access to bathroom facilities, causing the child to actively try to prevent bowel movements. The fact that the child previously had control indicates a behavioral cause. It is not necessary to refer to a specialist. Putting the child in diapers and resuming toilet training later is useful for the child who is unable to toilet train. Without further evaluation, the PNP cannot determine that there is a developmental delay. 10. The primary care pediatric nurse practitioner is concerned that a toddler may have vesicoureteral reflux based on a history of dysfunctional voiding patterns and a series of urinary tract infections. Which intervention is appropriate? a. Initiating a bladder retraining program b. Ordering a voiding cystourethrogram c. Referral to a urologist for evaluation d. Treatment with prophylactic antibiotics - Correct answer ANS: C If symptomatic vesicoureteral reflux is suspected, the PNP should refer the child to a urologist for diagnosis and initiation of treatment. A bladder retraining program does not treat the underlying cause. The urologist must order the VCUG and will decide if prophylactic antibiotics are indicated 11. The primary care pediatric nurse practitioner is counseling the parent of an 8-year-old child who has primary nocturnal enuresis. The nurse practitioner recommends an enuresis alarm, but the parent wishes to use medication. What will the nurse practitioner tell the parent? a. Anticholinergic medications are most commonly used for enuresis. P a g e 31 | 134 b. Drug therapy is an effective way to achieve long-term control. c. Drug therapy is safest when the nasal spray form is used. d. The combination of alarm therapy and intermittent drug therapy is best. - Correct answer ANS: D Drug therapy in combination with alarm therapy is effective, but parents must be cautioned to use drug therapy only when the child must be dry at night, such as when on a sleepover. Anticholinergic medications can cause constipation, which may exacerbate the problem and are not recommended as first- line treatments. Drug therapy alone can result in relapse of nocturnal enuresis when the drug is withdrawn and is not effective for long-term control. The nasal spray form of desmopressin has a black box warning because of a risk of hyponatremia. 1. The primary care pediatric nurse practitioner is performing a well child exam on a 4-month-old infant who is nursing exclusively. The mother reports that the infant has had a marked decrease in the number of stools each day, from 3 to 5 stools each day to only one stool every other day. How will the nurse practitioner respond? a. Ask the mother to describe the color and consistency of the stools. b. Explain to the mother that breastfed infants should have daily stools. c. Recommend using a glycerin suppository as needed. d. Suggest to the mother that she increase her intake of fluids. - Correct answer ANS: A It is common for older breastfed infants to stool less frequently. The PNP should assess the color and consistency of the stools to make sure that they are normal. As long as infants are happy, thriving, and free from clinical signs of GI distress, parents can be reassured that this is normal. It is not necessary to use a glycerin suppository or ask the mother to increase fluids in the absence of clinical pathology. 2. The primary care pediatric nurse practitioner is evaluating a 5-year-old child who has frequent soiling of stool associated with stomach aches and decreased appetite for the past 2 months. The parent states that the child has two or fewer formed bowel movements each week and has been toilet trained for about 2 years. Which initial assessment will the nurse practitioner make? a. History of neurogenic conditions b. Recent adjustments in the family c. Recent illnesses, fluid intake, changes in diet d. Toilet training history - Correct answer ANS: C The child has been toilet trained and has recently developed chronic constipation. The first step is to evaluate recent illnesses or dietary changes that could cause constipation and painful stools that resulted in stool withholding. The other options represent more underlying physiologic or psychological pathology and should be explored if simple physiologic causes are not present. 1. 1. The primary care pediatric nurse practitioner is evaluating a school-age child who, after removal of a pituitary tumor, has altered hypothalamic control over hunger and satiety. The child is morbidly obese and expresses feeling depressed because of the obesity. What will the nurse practitioner recommend? a. a. Developing a system to reward compliance with a dietary regimen b. b. Restricting all access to food in the house and at school c. c. Suggesting an after-school exercise program to help with weight loss d. d. Using a food diary to track all calories and food intake - Correct answer ANS: B Children with brain dysfunction affecting hypothalamic control may need rigid restrictions of access to all foods in refrigerators, cupboards, and even garbage cans parents may need to install locks to prevent access. Because the child has no ability to control hunger, developing a reward system is likely to fail. An exercise program and a food diary may be beneficial, but the primary concern is to restrict access to foods. - Correct answer P a g e 32 | 134 missed and if the child needs an MV supplement. MVs are not usually necessary and iron is not given unless there is a deficiency. It is not necessary to meet DRIs every day. Supplements may be necessary after data is collected. 1. 12. When counseling an adolescent with a family history of hyperinsulinemia and type 2 diabetes, the primary care pediatric nurse practitioner will recommend avoiding a. a. baked potato chips. b. b. canned vegetables. c. c. high-fiber cereals. d. d. processed breads. - Correct answer ANS: D High-glycemic foods, such as soda, sweetened juices, and processed breads, pastries, and crackers are more quickly converted to serum glucose and stimulate a sharp rise in insulin production and a subsequent rapid shift into hypoglycemia. To help prevent this in a child with a family history of this disorder, the PNP should recommend avoiding processed breads, pastries, and crackers. High levels of fructose and low fiber intake also contributes to this phenomenon. Baked potato chips, canned vegetables, and high-fiber cereals do not contribute to excess insulin production. 1. The primary care pediatric nurse practitioner is performing a pre-participation sports physical examination on a 14-year-old male who will be on the wrestling team at school. What will the nurse practitioner include when discussing healthy practices with this adolescent? a. Risks associated with repeatedly losing and gaining weight b. The need for an electrocardiogram or echocardiogram prior to participation c. The need to consume 20 to 30 grams of protein after exercise d. To consume water with CHO prior to activity lasting up to an hour - Correct answer ANS: A Wrestlers often try to lose weight rapidly prior to wrestling matches to put themselves into a lower weight category. It is important to teach young athletes about the risks associated with repeated weight loss and gain. ECG and echocardiograms are not recommended as a requirement for all pre-participation physical exams unless there is an indication for doing so, such as with syncope or murmurs. Athletes do not need to consume 10 to 20 grams of protein after exercise complex carbohydrates are recommended to improve muscle glycogen resynthesis. Plain water is recommended before, during, and after all activity lasting up to an hour. - Correct answer 2. The primary care pediatric nurse practitioner counseling the parent of an overweight school- age child about improving overall fitness. What will the nurse practitioner include? a. Encourage the child to begin by engaging in swimming or cycling. b. Exercise will help lower total cholesterol and low-density lipoproteins. c. School-age children need 60 minutes of moderate exercise daily. d. Strength training exercises are not safe for school-age children. - Correct answer ANS: A The AAP suggests that overweight children initially participate in activities that place less stress on weight-bearing joints, such as swimming or cycling. Exercise helps raise HDL levels but does not reduce total cholesterol or LDL levels. School-age children need 60 minutes of physical activity but not necessarily exercise each day. Strength training exercises are safe, but powerlifting and maximal weight training are not, because of effects on developing bones. 3. The parents of a pre-pubertal female who is on the local swim team tell the primary care pediatric nurse practitioner that their daughter wants to begin a strength training program to help improve her swimming ability. What will the nurse practitioner recommend? a. Avoiding strength training programs until after puberty to minimize the risk for injury b. Enrolling their daughter in a program that uses fixed weight machines or resistance bands c. Having their daughter participate in weight training 4 or 5 times each week for maximum effect d. Making sure that their daughter begins with the greatest weight tolerable using lower repetitions - Correct answer ANS: B P a g e 35 | 134 Fixed weights or resistance bands are recommended for pre-pubertal youth to help prevent injury. Strength training prior to menarche helps to strengthen long bones and is considered beneficial. Weight training should be 2 to 3 times weekly with a day in between sessions. Initially, youth should begin with a low number of sets and low intensity. 4. The parent of a 14-year-old child asks the primary care pediatric nurse practitioner how to help the child prevent injuries when basketball tryouts begin later in the school year. Which recommendation will be of most benefit? a. Preseason conditioning b. Proper footwear c. Protective knee braces d. Stretching before practices - Correct answer ANS: A Conditioning in the preseason is one of the most important things children can do to build muscle strength, to prevent sports injuries, and to learn how to make twisting, jumping, and landing movements safely. Proper footwear is also recommended but is not the most important. Protective knee braces may be worn but do not prevent injury. Stretching should be done after warming up to maintain flexibility. 5. The parent of a high school basketball player tells the primary care pediatric nurse practitioner that the adolescent becomes short of breath only when exercising. What will the nurse practitioner recommend? a. Permanent discontinuation of all strenuous and aerobic activities b. Enrollment in a conditioning program to improve performance c. Evaluation for underlying cardiac causes of this symptom d. Treatment for exercise-induced asthma with a bronchodilator - Correct answer ANS: C While shortness of breath may indicate several more benign causes, athletes who exhibit this symptom should be evaluated for underlying cardiac causes to prevent sudden cardiac death. Once this is ruled out, other causes may be considered, such as EIA or poor conditioning. 6. A 15-year-old female basketball player who has secondary amenorrhea is evaluated by the primary care pediatric nurse practitioner who notes a BMI in the 3rd percentile. What will the nurse practitioner counsel this patient? a. That amenorrhea in female athletes is not concerning b. That she should begin a program of plyometrics and strength trainin c. To consider a different sport, such as volleyball d. To work with a dietician to improve healthy weight gain - Correct answer ANS: D Female athletes who have amenorrhea have an increased risk of stress fractures. The adolescent should work to attain a healthy weight, which should allow normal periods to return and reduce this risk. Even though amenorrhea in female athletes is common, it is concerning. Plyometrics and volleyball can increase the risk of stress fractures since both involve jumping and thus not be suggested. 7. The parent of a child who has asthma asks the primary care pediatric nurse practitioner about whether the child may engage in strenuous exercise. What will the nurse practitioner tell the parent? a. Children with asthma should be excluded from vigorous exercise and most strenuous sports. b. Children with asthma show improved aerobic and anaerobic fitness with moderate to vigorous/physical activity. c. Physical activity has been shown to improve overall pulmonary function in children with asthma. d. Vigorous exercise helps improve symptoms in children with poorly controlled asthma. - Correct answer ANS: B Children with mild or well-controlled asthma may participate in moderate to vigorous sports and show P a g e 36 | 134 benefits to aerobic and anaerobic fitness, which helps lung function and overall health outcomes. It is not necessary to exclude children with asthma from sports as long as symptoms are well controlled. Overall pulmonary function does not substantially improve with exercise. Children with poor control should not engage in sports until symptoms are under control. 8. The primary care pediatric nurse practitioner diagnoses a high school basketball player with mononucleosis. The adolescent asks when she may resume play. What will the nurse practitioner tell her? a. After 3 weeks, she may begin lifting weights but not full sports. b. After 4 weeks, she may return to full play and practice. c. At 4 weeks, she must have an exam to determine fitness for play. d. She may engage in moderate exertion and practice after 3 weeks. - Correct answer ANS: C Full return to play should be determined on a case-by-case basis and is generally considered safe at 4 weeks after symptom onset, assuming physical stamina has returned, all symptoms have resolved, and the sport does not increase intraabdominal pressure during play. Athletes should avoid any form of exertion, including all sports during the first 3 weeks at a minimum and should avoid anything with a risk of chest or abdominal contact or anything that involves increased intra-abdominal pressure. Splenic rupture can occur spontaneously (rare), but the risk of rupture increases when participating in a contact or collision sport or a sport in which there is an increase in intraabdominal pressure. The nurse practitioner should recommend an exam at 4 weeks to determine fitness for play. 9. The primary care pediatric nurse practitioner is counseling a parent about bicycle helmet use. The parent reports having a helmet used a year previously by an older child and wonders about using it for a younger child since they are so expensive. What will the nurse practitioner tell the parent? a. "As long as the helmet does not have cracks, you may use it." b. "If the helmet is free from marks, you may use it." c. "You may continue to use a helmet up to 10 years." d. "You should always purchase a new helmet for each child." - Correct answer ANS: B While parents should be taught not to purchase a secondhand helmet, using a fairly new, undamaged helmet from an older child is acceptable. Any helmet that has marks should be discarded, even if not cracked. Helmets should be replaced every 5 years or sooner, depending on the manufacturer's recommendations. It is not necessary to purchase a new helmet for each child, especially if money is an issue. 10. The primary care pediatric nurse practitioner is performing a well child examination on a high school age adolescent who plays football who has hypercalciuria. Which dietary supplement will the nurse practitioner question the adolescent about? a. Protein supplements b. Salt tablets c. Sports drinks d. Vitamin C - Correct answer ANS: A Protein supplements can cause hypercalciuria with calcium loss and dehydration if protein intake is too high. Salt tablets can cause hypernatremia and delayed gastric emptying. Sports drinks are high in sugar and electrolytes, which will not affect the calcium content of the urine. It is not necessary to take vitamin C. P a g e 37 | 134 Vocal cord dysfunction causes shortness of breath and must be managed but does not prevent children from participation in sports. It does not indicate underlying cardiac problems and does not mean children should avoid any sport that may increase heart or respiratory rates. 19. The primary care pediatric nurse practitioner is offering anticipatory guidance to the parents of a 6- year-old child who has Down syndrome. What will the nurse practitioner tell the parents about physical activity and sports in school? a. Children with Down syndrome get frustrated easily when engaging in sports. b. Children with Down syndrome should not participate in strenuous aerobic activity. c. Their child should have a cervical spine evaluation before participation in sports. d. Their child should only participate in sports sanctioned by the Special Olympics. - Correct answer ANS: C Because up to 40% of children with Down syndrome have a hypermobility or instability between C1-C2 and up to 61% have occipito-atlantal hypermobility, they should undergo radiological evaluation of the cervical spine to be cleared for strenuous sports. Many children and adolescents with intellectual and developmental disabilities (including those with Down, fragile X, Turner, or Klinefelter syndromes or autism) are capable of performing exercise or strenuous activities. Special needs children should be encouraged to participate in sports to increase physical abilities and increase self-confidence. Children with Down syndrome may benefit from strenuous aerobic activity and may participate in any sports once cervical spine stability is evaluated, not just those sanctioned by the Special Olympics. 1. The parent of a 3-year-old child tells the primary care pediatric nurse practitioner that after falling asleep in the living room and being awakened to go to bed one evening, the child appeared confused and disoriented for a period of time. What will the nurse practitioner counsel this parent? a. That if this occurs again, to question the child about nightmares b. That this is a sign of sleep walking and could be dangerous c. That this is a type of sleep terror which will resolve over time d. That this is probably a benign, temporary type of a sleep disorder - Correct answer ANS: D This child most likely exhibits confusional arousal, which occurs when a child is awakened from a deep sleep during the first part of the night. It is most likely benign and temporary, usually diminishing by age 5 years. It is not a sign of nightmares or night terrors. It may be the start of sleep walking but is less likely. 2. The primary care pediatric nurse practitioner is counseling the parents of a toddler about sleep. The parents report that the toddler has recently begun resisting sleep and is often more irritable during the day. What will the nurse practitioner recommend? a. Co-sleeping with the child to help alleviate possible nighttime fears b. Referral to a sleep disorders clinic for evaluation of sleep-disordered breathing c. Reintroducing a second, morning nap time to compensate for lost sleep d. Understanding that sleep resistance is a common developmental problem - Correct answer ANS: D Toddlers may develop sleep resistance as a normal part of their behaviors associated with increased autonomy or may have nighttime fears or night terrors. Parents should understand that this is common and transient. Co-sleeping may be practiced in some cultures but is not recommended. It is not necessary to refer to a sleep disorders clinic unless there are specific symptoms, such as snoring or restless sleep or sleepiness in spite of adequate sleep. P a g e 40 | 134 3. During a well child examination, the primary care pediatric nurse practitioner learns that a 5- year-old child has had several episodes of walking out of the bedroom after falling asleep, looking dazed, with open eyes, and saying things that don't make sense. What will the nurse practitioner recommend? a. Establishing a graduated extinction program and good sleep hygiene b. Making sure that stairs are blocked and doors are locked c. Referral to a sleep disorder clinic for evaluation of a parasomnia d. To awaken the child when these occur and asking about nightmares - Correct answer ANS: B Parents of children with sleep walking should be assured that this is relatively benign but should make sure the house is secure so the child will not cause self-harm. Graduated extinction and sleep hygiene are used for children who have difficulty initiating or maintaining sleep. Referral to a sleep disorder clinic may be warranted if the child has an episode of leaving the house or some other dangerous activity. The child should be guided back to bed without awakening. 4. The parent of a 4-year-old who has difficulty initiating and maintaining sleep has tried several nonpharmacological methods with variable success and asks about medications. What will the primary care pediatric nurse practitioner recommend? a. Diphenhydramine b. Lorazepam c. Melatonin d. Zolpidem - Correct answer ANS: C Medications to treat dyssomnias are generally discouraged in children, since they have side effects and since the mainstay of treatment is behavioral therapy and sleep hygiene. If medications are used, melatonin is the most commonly prescribed. Diphenhydramine can lead to parasomnias in some children. Benzodiazepines, such as lorazepam, can cause dependence. Sedatives, such as zolpidem, have high levels of side effects. 5. A child with Down syndrome who has sleep-disordered breathing with obstructive sleep apnea continues to have symptoms in spite of tonsillectomy and adenoidectomy and treatment with a leukotriene receptor antagonist medication and a nasal steroid spray. The primary care pediatric nurse practitioner will refer the child to a sleep disorder clinic to discuss which therapy? a. Craniofacial surgery b. Oral appliances c. Positive airway pressure therapy d. Supplemental oxygen - Correct answer ANS: C Positive airway pressure therapy can be used to treat sleep-disordered breathing in children who have failed other therapies and even developmentally delayed children show improvement in behaviors after this therapy. Craniofacial surgery may be used in the presence of maxillofacial deformities that affect sleep-disordered breathing but is a last option. Oral appliances may be used if deformities can be corrected in this manner. Supplemental oxygen helps with oxygen saturations but not with disordered breathing patterns themselves. 6. The primary care pediatric nurse practitioner is counseling a new parent about ways to reduce the risk of sudden infant death syndrome (SIDS). What will the nurse practitioner include when discussing SIDS? a. Bed-sharing with infants greatly increases the risk of SIDS. b. Breastfeeding does not appear to have any influence on SIDS risk. c. Infants who attend day care have a higher than usual incidence of SIDS. P a g e 41 | 134 d. There is no difference in SIDS rates in immunized versus non-immunized infants. - Correct answer ANS: A Bed-sharing with infants has been shown to have a five-fold increase in the incidence of SIDS, even with non-drug using and non-smoking parents smoking, alcohol, and drug use increase this risk even further. Breastfeeding is recommended and is associated with a reduced risk of SIDS. Day care is not mentioned as increasing SIDS risk. Infants who are immunized have a 50% reduction in SIDS risk, according to research evidence. - Correct answer 7. The primary care pediatric nurse practitioner is performing a well baby examination on a 2- week-old infant. The parent is concerned that the infant sleeps too much. The nurse practitioner asks the parent to keep a sleep log and will teach the parent that which amount of sleep per day is optimal for this infant? a. 10 to 12 hours b. 12 to 15 hours c. 15 to 18 hours d. 18 to 20 hours - Correct answer ANS: B Newborns sleep a total of 15 to 18 hours per day. 8. The parent of a school-age child who is overweight tells the primary care pediatric nurse practitioner that the child seems to crave high-calorie, high-carbohydrate foods, even when full. The nurse practitioner learns that the child is often irritable and sleepy at school in spite of sleeping 9 or 10 hours each night. What will the nurse practitioner recommend? a. Assessment of leptin and ghrelin hormone levels b. Consultation with a dietician to develop an appropriate diet c. Referral to a sleep disorder clinic for a sleep study d. Taking one or two naps each day to increase the amount of sleep - Correct answer ANS: C Obstructive sleep apnea has been suggested to be a contributing factor to the pathogenesis of obesity by inducing leptin resistance and increasing ghrelin levels, two hormones that regulate satiety. The child shows symptoms of these abnormalities by craving high-calorie comfort foods. The child should be evaluated for this underlying cause. Assessment of these hormone levels is not routinely done. Consultation with a dietician may be necessary at some point but does not get at the underlying problem. Increasing sleep time with naps has not been shown to counteract the obesity effect. 9. The parent of a school-age child tells the primary care pediatric nurse practitioner that the child is restless most nights and complains often that bugs are in the bed. After consultation with a sleep disorder specialist and subsequent evaluation of a ferritin level of 30, the nurse practitioner may expect to treat this child with a. clonazepam. b. ferrous sulfate. c. gabapentin. d. sertraline. - Correct answer ANS: B A ferritin level of less than 50 is associated with periodic limb movements. The treatment for this is ferrous sulfate 3 mg/kg per day. Clonazepam and gabapentin may be ordered if ferritin levels are normal and other organic causes of PLM have been ruled out. Sertraline may make PLM worse. 10. An adolescent exhibits mild depressive symptoms and tells the primary care pediatric nurse practitioner that he is most concerned about difficulty falling and staying asleep. The adolescent does not want to take medication to treat the depressive symptoms. What will the nurse practitioner recommend? a. A program of sleep hygiene and gradual sleep extension P a g e 42 | 134 5. During a well child examination of a 6-year-old girl, the primary care pediatric nurse practitioner notes that the child becomes embarrassed and resists taking off her underwear for the exam. What should the nurse practitioner infer from this observation? a. The child has been sexually molested. b. The child is feeling violated by the examiner. c. The parent is exhibiting regressive behavior. d. This is a normal reaction in a child of this age. - Correct answer ANS: D Young school-age children can be extremely modest and embarrassed and resist taking off their clothes for an examiner. Since this is normal, it does not indicate a history of sexual abuse unless other signs are present. Older school-age children more commonly feel violated during an exam, not younger children. This response of increased modesty is age-appropriate and not regressive. 6. The primary care pediatric nurse practitioner is providing anticipatory guidance to the parent of a school-age boy. The parent expresses concerns that the child prefers to play with dolls, is worried that the child will be a homosexual, and asks what can be done to prevent this from happening. What will the nurse practitioner tell this parent? a. Homosexual identity formation cannot be predicted by early childhood behavior. b. Masculinizing boys from an early age helps to determine heterosexual orientation. c. Sexual orientation identification begins late in adolescence and not in childhood. d. The development of sexual orientation is generally a multifaceted process. - Correct answer ANS: D The etiology and age of preferred sexual orientation is unknown, and the sequential developmental signs are debated. The development of sexual orientation is most likely multifaceted and cannot be predicted by one phenomenon, such as playing with dolls alone. Early childhood behavior can predict homosexual orientation as girls may feel "unfeminine" and boys may exhibit feminine tendencies. It is clear that psychosocial components and parenting do not cause or prevent homosexuality. 7. The primary care pediatric nurse practitioner is performing a well child examination on a 3- year-old. The child's parent reports that the child has recently begun masturbating. What will the nurse practitioner counsel this parent? a. To allow the behavior whenever it occurs, since it is normal b. To discuss sexuality with the child c. To explore whether the child is being abused d. To teach the child about privacy and hand hygiene ANS: D Masturbation is normal at this age and children do this because it is pleasurable. Parents should be taught to discuss privacy and hygiene with the child and to encourage the child to limit the activity to a private place. At this age, the behavior is not associated with sexual fantasies, so a discussion of sexuality is not warranted. Masturbation at this age is common and is not usually an indication of abuse. 8. The parent of an 8-year-old child tells the primary care pediatric nurse practitioner that the child has begun to ask questions about why a schoolmate has "2 daddies" and wonders how to talk to the child about this. What will the nurse practitioner recommend? a. Beginning a discussion about different types of sexual relationships and same-sex partners b. Discussing the issue with the child in terms of the parent's religious values and norms c. Explaining that not all families are the same and what is most important is that they love and care for their children d. Telling the child that some adult relationships are complicated and will be understood when the P a g e 45 | 134 child is older - Correct answer ANS: C School age is a good time for parents to reinforce the notion that there is diversity in families within which parents and adults love and care for their children. It is not necessary to be explicit but to establish a good history of communication and to explain complex issues to children at a level of the child's understanding. In this way, the child will know that parents are accessible and open to discussion of complex and/or puzzling issues. 9. During a well child examination, a 15-year-old female tells the primary care pediatric nurse practitioner that some of her friends have begun having sex. She has a boyfriend but denies engaging in sex with him. What will the nurse practitioner do initially? a. Ask her for her definitions of "sex." b. Discuss the risks of sexually transmitted diseases. c. Find out if she is considering sexual relations. d. Give her information about contraception. - Correct answer ANS: A Many adolescents do not equate oral or anal intercourse with sex, so it is important to find out how this patient defines sex. The other options also may be considered depending on the situation, however, clarity about the words used in the discussion are most important initially for the nurse practitioner to focus the subsequent conversation appropriately. 1. 1. The mother of a 15-year-old female expresses concerns that her daughter may be sexually active because she's had a steady boyfriend for over a year. The primary care pediatric nurse practitioner learns that the family is Catholic and that the mother had an abortion when she was 16 years old. What will the nurse practitioner do initially? a. a. Explore the mother's feelings about her own past experience. b. b. Offer to prescribe contraception to prevent pregnancy. c. c. Recommend that the mother discuss this with her daughter. d. d. Suggest that the mother talk to a priest about her daughter. - Correct answer ANS: A An initial step when assisting families to manage ethical and behavioral issues is to assist parents and children in values clarification. Before offering other suggestions, the PNP should help the mother to clarify her own feelings. The other options may be necessary after values are clarified. 1. 2. The primary care pediatric nurse practitioner sees a 6-year-old child after a hospitalization for injuries sustained in a motor vehicle accident (MVA) in which the child's grandfather was killed. The parent states that it is difficult to get the child to stop talking about the accident and is worried that the child will have permanent emotional scars. What will the nurse practitioner suggest? a. a. Assure the child that he is safe and this won't happen again. b. b. Encourage the child to express and examine feelings. c. c. Reassure the child that his grandfather is in heaven. d. d. Redirect these conversations to happier topics. - Correct answer ANS: B Management goals when health crises occur should focus on helping children and families make sense of the events and to better understand what is happening to help regain control. Helping the child to express and examine feelings will help him to put what happened in perspective. Assuring the child that accidents won't happen again is false and can lead to even more distress in the future. Telling the child that the grandparent is in heaven doesn't address all his fears about the accident. Redirecting the conversation to happier topics minimizes the child's concerns. 1. 3. The primary care pediatric nurse is performing a well child examination on an adolescent who was adopted as a toddler. The parent reports that the child had been removed from an abusive home at age 3 years. What will the nurse practitioner evaluate in light of possible long-term effects of this early situation? a. a. Cognitive and psychosocial development b. b. Mental health and suicide risk c. c. Moral development and conscience formation d. d. Spirituality, faith, and religious affiliation - Correct answer ANS: B Findings of early research suggest that epigenetic changes may mediate the effect of prenatal and infant environment on mental health and disease in older children and adults. Adult suicide victims who were abused as children show decreased levels of a genetic marker of a stress P a g e 46 | 134 reduction gene, suggesting that child abuse may have an epigenetic effect leading to prolonged stress and mental disorders. 1. 4. During a well child examination of a school-age child from a family who recently immigrated from Africa, the primary care pediatric nurse practitioner learns that the child has been involved in many arguments at school. The parents are concerned that their child will never fit in with classmates. How will the nurse practitioner address this situation? a. a. Assess the conditions in the country of origin prior to immigration. b. b. Recommend counseling to determine underlying causes of this behavior. c. c. Stress that this may be a normal response to feeling different at school. d. d. Suggest that the child may be responding to being bullied by others. - Correct answer ANS: A Many children who exhibit signs of spiritual distress can express this by being angry or withdrawn. Trauma or violence to the self or to others can contribute to spiritual distress. The PNP should evaluate conditions in the country of origin since it is likely that the family may have escaped war or persecution prior to immigration. Once underlying conditions are known, the PNP can recommend options or try to explain the behavior to parents. 1. 5. While the primary care pediatric nurse practitioner is discussing anticipatory guidance with the mother of a 12 month old, the child repeatedly pulls objects out of the mother's purse. Each time, the mother slaps the child's hands as she takes the objects away. What will the nurse practitioner recommend to help the mother manage this child's misbehavior in a developmentally appropriate manner? a. a. Keep her purse up high and out of the child's reach. b. b. Place acceptable objects in her purse for the child to find. c. c. Say "No!" instead of slapping the child's hands. d. d. Use timeout each time the child gets into the purse. - Correct answer ANS: A Parents should provide a developmentally appropriate environment to minimize children's misbehavior. Children at this age who are naturally curious will explore the environment and will seek out objects within their reach. It is easier to put the purse up high than to have to repeatedly say "No." Putting acceptable objects in her purse only reinforces the undesired behavior. 1. The primary care pediatric nurse practitioner is performing an examination on a 2-year-old child who has been placed in emergency foster care with a grandparent after the child's mother has been arrested for drug use. The child has a history of asthma with frequent exacerbations because of parental smoking. What is a priority for the nurse practitioner at this visit? a. Evaluation of financial resources, medical insurance, and access to health care and medications b. Providing a list of websites and community-based support groups for grandparents parenting grandchildren c. Referral to a social worker to help the child deal with emotional conflict related to separation from the parent d. Teaching the grandparent about the need for consistency in routines and discipline for the child - Correct answer ANS: A A high percentage of grandparents who parent grandchildren have financial difficulties and most cannot claim grandchildren as dependents for health care. This child has a chronic disease and will need medication and possibly hospitalization, so the PNP should assess resources and access to care. The other options are important but are not a priority in the initial visit. 2. Adolescent children are more likely to smoke cigarettes and drink alcohol if they live with a. cohabitating parents. b. grandparents. c. homosexual parents. d. single parents. - Correct answer ANS: A Adolescents living in cohabitating households are more likely to smoke and drink in comparison with adolescents living in married or single parent households. Children living with homosexual parents and children living with grandparents do not have this increased risk. P a g e 47 | 134 c. Performing a colposcopic examination to evaluate for trauma d. Referring the child to the ED for forensic specimen collection - Correct answer ANS: D If sexual abuse has occurred within 72 hours, it is required that appropriate forensic specimens be collected. Getting a history from the child is part of the child abuse evaluation and will be done by the child abuse team, as well as obtaining urethral specimens for STI. Colposcopic exams should be done by an expert in sexual abuse if trauma is suspected but is not performed by the PCPNP. 11. The primary care pediatric nurse practitioner is performing a well baby examination on a newborn whose mother is 17 years old. The mother states that she is living with her parents and plans to finish high school. The maternal grandmother will care for the infant while she is in school. What will the nurse practitioner discuss with this mother at this visit? a. Early child intervention programs b. Her needs for socialization with peers c. Immunizations and well child visits d. Referral to a community health nurse - Correct answer ANS: B Adolescent mothers face problems inherent when this major role is assumed before they are developmentally ready themselves. Their developmental needs may sometimes be in conflict with their children. Although this mother has support from her family, the PNP should assess her desires to socialize with her peers to evaluate her developmental needs. Early child intervention may be necessary when the child is preschool age and immunizations and well child visits should be discussed, but these do not take precedence over the need to evaluate a potential role conflict. Referrals to community health may be necessary if problems arise. 12. The primary care pediatric nurse practitioner is examining a young child who has cerebral palsy. Which part of the family history raises concerns about potential child maltreatment? a. Child attends day care b. Limited financial resources c. Mother works outside the home d. No membership in a church - Correct answer ANS: B Limited financial resources can put a strain on caring for a child with special needs whose medical needs are expensive. The fact that the mother works outside the home and the child attends day care may actually provide some respite from the strain of caring for a special needs child. Families may have strong spiritual values whether they attend church or not. 13. The mother of two school-age children tells the primary care pediatric nurse practitioner that she and the children's father are divorcing and asks for advice to help the children cope with the situation. The nurse practitioner will counsel her to a. allow visitation only on weekends. b. maintain her own social life. c. notify the children's teachers. d. use a social support network. - Correct answer ANS: D The availability of a social support system is a key factor in coping with divorce. Visitation patterns should be consistent and collaborative but not necessarily only on certain days. Although the mother should eventually maintain a social life, it is not a key early on to helping children cope with a divorce. Notifying teachers does not necessarily help children to cope. 14. The primary care pediatric nurse practitioner is examining an infant who has otitis media and learns P a g e 50 | 134 that the mother and child are homeless. Besides assisting the mother to obtain medication to treat this illness, what is a priority during this visit? a. Assisting the mother to obtain transportation for health care needs b. Determining well child examination history and immunization status c. Making sure the family has access to WIC and food stamps resources d. Obtaining a tuberculosis skin test and scheduling a return office visit - Correct answer ANS: B Even if a health care visit is in response to a crisis that cannot be denied, assessment of homeless patients should include well child care and immunizations on the operating principle that every child should receive the maximum health care possible. The other options may be part of ongoing assistance and evaluation of homeless families, but the initial response should be evaluation of well child needs. 15. The primary care pediatric nurse practitioner is discussing newborn care with a mother who is pregnant with triplets. When counseling the mother about feeding issues, the nurse practitioner will recommend a. developing a plan to rotate breastfeeding for her infants. b. making sure that the triplets are on the same feeding schedule. c. pumping her breasts so she can feed breastmilk to all three. d. supplementing with formula to ensure adequate nutrition. - Correct answer ANS: A Mothers of multiples can breastfeed their infants. With more than two infants, the mother should develop a rotation schedule. While it is recommended to attempt to get the babies on the same schedules for feeding and sleeping, this is not always possible and not necessary. Infant suckling is the best way to increase milk supply. Supplementation is not recommended unless her milk supply is inadequate. 1. A parent who encourages competitiveness in a child who excels at a single sport but not in others may also encourage a sense of a. competence. b. insecurity. c. significance. d. worthiness. - Correct answer ANS: B Children who gain praise for external measures, such as performance of a sport, may end up unduly comparing themselves with others and feel insecure, inferior, and inadequate, even as they continue to excel in this sport. Competence comes from feeling capable and confident and able to approach new tasks. Significance comes from having a sense of belonging and being accepted unconditionally this child's self-worth is dependent on performance in a sport. Worthiness is based on an understanding of having a purpose in life and is also unconditional. - Correct answer 2. The parent of a 15-year-old male is concerned that he refuses to eat meals with the family and consumes only protein drinks. The adolescent is on the track team at school and spends much of his time training and working out. The primary care pediatric nurse practitioner notes that his weight and BMI have dropped from the 20th percentile to the 3rd percentile in the past year. This child most likely has a problem with a. body image. b. personal identity. c. role performance. d. self-esteem. - Correct answer ANS: A Children with disturbed body image may have concerns related to appearance, body size, function, or P a g e 51 | 134 potential. Possible behaviors include eating disorders and a preoccupation with the perfect body. Children with body image problems become overly concerned with appearance and compare themselves to others. This child is losing weight and working out too often to try to change his body. Children with personal identity issues internalize negative perceptions of others and manifest feelings of inferiority. Children with role performance problems feel incompetent and are hesitant to try new things or become perfectionists to overcompensate. Children with poor self-esteem seek attention, importance, and security. 3. The primary care pediatric nurse practitioner is evaluating a 16-year-old adolescent male who is on the high school wrestling team and whose weight fluctuates as much as 7 or 8 pounds before matches. The child is eager to talk about the various trophies he has won. When he expresses confidence that he will get a wrestling scholarship for college, his father remarks that his grades will never be good enough for college, causing him to blame his teachers. The nurse practitioner may identify potential problems with a. body image. b. personal identity. c. role performance. d. self-esteem. - Correct answer ANS: D Children with poor self-esteem seek attention, importance, and security and may become self- absorbed with external markers of self-worth, such as performance in a sport. Another mark of insecurity is defensiveness, which this child exhibits by blaming his teachers for his poor grades. Children with body image problems become overly concerned with appearance and compare themselves to others. This child is losing and gaining weight to be better at wrestling, not to look different. Children with personal identity issues internalize negative perceptions of others and manifest feelings of inferiority. Children with role performance problems feel incompetent and are hesitant to try new things or become perfectionists to overcompensate. 4. A school-age child enjoys playing basketball but doesn't make the intramural team. Which response by the child is characteristic of the concept of a growth mindset? a. "I didn't play well on the day of the tryouts." b. "I'll just have to find another sport I'm good at." c. "I'll need to work more on my outside shot." d. "I'm probably too short to be really good at this sport." - Correct answer ANS: C Children who have a growth mindset have been taught to believe that hard work is key to success and that effort and practice contribute to growth. By not being discouraged and identifying something to work on, the child is exhibiting a growth mindset. Saying that he didn't play well is making excuses, while stating he needs to find something else he's good at or blaming his failure on a physical characteristic indicates a belief that success is dependent on fixed traits. 5. The primary care pediatric nurse practitioner is performing a well child examination on a fussy toddler who has red hair. The child's parent tells the toddler to stop being fussy and says, "red hair gives him such a temper." Which common error that erodes self-esteem is this? a. Dwelling on negatives b. Expecting too much c. Negating the child's feelings d. Stereotyping and typecasting - Correct answer ANS: D P a g e 52 | 134 like symptoms. If the child does have separation anxiety disorder (SAD), treatment and not discipline is warranted. 6. An adolescent has recently begun doing poorly in school and has stopped participating in sports and other extracurricular activities. During the history interview, the adolescent reports feeling tired, having difficulty concentrating, and experiencing a loss of appetite for the past few weeks but cannot attribute these changes to any major life event. Which is an important next step in managing this patient? a. Administering a diagnostic rating scale for depression b. Considering a short-term trial of an antidepressant medication c. Determining suicidal ideation and risk of suicide d. Referring the adolescent to a mental health specialist - Correct answer ANS: C Because this adolescent exhibits clear signs of depression, the first goals of management are to determine suicidal risk and to intervene to prevent suicide since the risk of suicide is greatest during the first 4 weeks of a depressive episode. A diagnostic rating scale may help in diagnosing the depression, but assessing suicide risk is a priority. Antidepressant medications may be useful but are best initiated by a mental health specialist. The initial response in adolescents should be to determine suicide risk to decide whether to admit to inpatient therapy or refer to a mental health specialist. 7. A child has a difficult temperament. What will the primary care pediatric nurse practitioner tell the parent about managing this child's behavior? a. A difficult temperament is its own risk factor for maladjustment disorders. b. Children with difficult temperaments need strict adherence to rules. c. Having a difficult temperament limits intelligence and emotional maturity d. It is important for the parent to learn to manage criticism and power struggles. - Correct answer ANS: D Children with difficult temperaments tend to engender parental criticism and irritability, power struggles, and restrictive parenting, and this dynamic leads to difficulties with psychosocial adjustment. Parents of children with difficult temperaments must be aware of behavioral manifestations as temperament expressions in order to reframe their own responses to these behaviors. A difficult temperament alone is not a risk factor for maladjustment. Restrictive parenting only increases power struggles and maladaptive behaviors. Temperament is unrelated to IQ but does affect academic outcomes because of behavior issues in school. 8. A toddler has begun hitting and biting other children at a day care center and is exhibiting temper tantrums and bad language at home. The parent reports that these behaviors began shortly after a sibling was born. What will the primary care pediatric nurse practitioner do? a. Advise the parent that the child is exhibiting early symptoms of ADHD. b. Engage the parent in positive parenting strategies to facilitate appropriate child coping. c. Recommend evaluating the child for conduct or oppositional defiant disorder. d. Suggest putting the child in another day care center to ameliorate the problems. - Correct answer ANS: B Children with social aggression may exhibit the behaviors described above. When social aggression is a response to acute stress, such as the birth of a sibling, the problem usually resolves if parents use positive parenting strategies and facilitate developmentally appropriate child coping efforts. These are not symptoms of ADHD. Conduct disorder (CD) symptoms usually manifest in the preschool age. Oppositional defiant disorder (ODD) is characterized by disobedience rather than aggressiveness. Moving P a g e 55 | 134 the child to another day care or school does not solve the problem. 9. The parent of a school-age child reports that the child doesn't like being alone in rooms because of a fear of aliens hiding in closets. What will the primary care pediatric nurse practitioner tell the parent? a. "Fear of imaginary creatures does not usually occur at this age." b. "I may need to refer your child to a pediatric mental health specialist." c. "Your child is expressing normal fears for a school-age child." d. "Your child may be watching too much violence on television." - Correct answer ANS: A Preschoolers tend to fear imaginary creatures but not school-age children. The PNP does not have enough information about the severity of the fear or its effects on behavior to refer the child to a mental health specialist. The PNP should evaluate the situation before attributing the fear to a cause such as television. 10. The parents of a 4-year-old boy are concerned because he has begun twisting and pulling out his hair, especially when he is tired or stressed. What will the primary care pediatric nurse practitioner recommend as part of an initial approach to treat this behavior? a. Consultation with a pediatric behavioral specialist b. Cutting his hair so that it is too short to pull c. Long-term anti-streptococcal prophylaxis d. Medication with risperidol or clonidine - Correct answer ANS: B Children with mild to moderate tic disorders can be managed with cognitive-behavioral therapy part of this includes strategies to help distinguish the undesired behavior, such as cutting the hair so it cannot be pulled out. If this is not effective, consultation with a behavioral specialist and medications, prescribed following assessment by a qualified mental health specialist, may be needed. - Correct answer 11. An adolescent is diagnosed with major depression, and the mental health specialist has prescribed fluoxetine. What other treatment is important to protect against suicide risk? a. Addition of risperidone therapy b. Cognitive-behavioral therapy c. Family therapy d. Hospitalization - Correct answer ANS: B Cognitive-behavioral therapy appears to have a protective effect against suicide and the best treatment responses come from combinations of cognitive-behavioral therapy and SSRIs. Risperidone and other antipsychotics are used if psychosis is present to control those symptoms. Family therapy is useful but does not add protection from suicide. Hospitalization is not the first-line treatment and is used for severe exacerbations or suicide attempts. 12. A middle-school-age child is skipping school frequently and getting poor grades since the child's father was killed while deployed in the military. How will the primary care pediatric nurse practitioner manage this situation? a. Prescribe short-term antidepressants for this situational depression. b. Refer the child to a mental health specialist for evaluation and treatment. c. Schedule extended appointments for counseling and mental health interventio. d. Suggest that the child have close follow-up by a school counselor. - Correct answer ANS: B Children who are experiencing enduring problems, such as the loss of a parent, should be treated either in consultation with or by referral to a pediatric mental health specialist. Antidepressants and other pharmacotherapeutic agents should never be used without a thorough mental health evaluation. The PNP is not qualified to manage this in a primary care setting without consultation or referral, nor is a school P a g e 56 | 134 counselor, although both may be part of the team of professionals who help to manage this child. 13. The parent of a school-age child reports that the child becomes frustrated when unable to perform tasks well and often has temper tantrums and difficulty sleeping. Which disorder may be considered in this child? a. Generalized anxiety disorder (GAD) b. Obsessive-compulsive disorder (OCD) c. Pediatric autoimmune neuropsychiatric disorder associated with streptococcal infection (PANDAS) d. Separation anxiety disorder (SAD) - Correct answer ANS: A GAD is characterized by over-concern about competence, significant self-consciousness, irritability and tantrums, and poor sleep. OCD results in recurring thoughts, images, or impulses. Patients with PANDAS have OCD- and Tourette-like symptoms. SAD causes difficulties separating from caregivers and being away from home. 14. A 14-year-old female comes to the clinic with amenorrhea for 3 months. A pregnancy test is negative. The adolescent's body weight is at 82% of expected for height and age. The mother reports that her daughter often throws up and refuses to eat most foods. Which condition does the primary care pediatric nurse practitioner suspect? a. Anorexia nervosa b. Bulimia nervosa c. Depression d. Substance abuse - Correct answer ANS: A Children with anorexia nervosa are usually underweight. Refusal to maintain body weight at least 85% expected for age and height or failure to gain weight during growth periods so that weight drops below 85% expected is one of the diagnostic criteria of anorexia. Some may throw up frequently, but children with bulimia nervosa are generally average weight or overweight. Depression and substance abuse may be comorbidities, but these signs are consistent with anorexia nervosa. 15. The parent of a school-age child is concerned because the child has started to express anger about a grandparent's death even though this occurred when the child was a toddler. What will the primary care pediatric nurse practitioner tell the parent? a. Anger is an abnormal reaction to bereavement and loss in this age child. b. Counseling is needed since the child has had sufficient time to resolve this issue. c. Grief and bereavement lasting longer than a year may require medication. d. The significance of this loss must be reworked at each developmental level. - Correct answer ANS: D At any given developmental stage, children resolve the effect of the death only at that developmental level and thus must rework the significance of the loss at each stage of development. Anger is a common reaction to loss in school-age children. Counseling and medication are not indicated since this is a normal response. 16. The primary care pediatric nurse practitioner attempts to learn more about the emotional health of an 18-month-old child through which assessment strategy? a. Asking the child to tell a story using dolls and other props b. Asking the child to draw a picture of him- or herself and other family members c. Interviewing the child separately from caretakers and parents d. Observation of the child with caretakers in structured and unstructured situations - Correct answer ANS: D P a g e 57 | 134 voluntarily studies a known or new drug in children. The FDAMA also requires pharmaceutical companies to survey existing data and determine potential drug use and indications in pediatric populations. 5. The primary care pediatric nurse practitioner is considering using a drug for an "off-label" use in a child. The nurse practitioner has used the drug in a similar situation previously, has consulted a pharmacology resource and the FDA website, and has determined that there are no significant contraindications and warnings for this child. What else must the nurse practitioner do when prescribing this drug? a. Discuss recommendations with the parents and document their consent. b. Document anecdotal reports of previous use of the drug by other providers. c. Follow up daily with the parents to determine safe administration of the drug. d. Report this use to the FDA Medwatch website for tracking purposes. - Correct answer ANS: A Many prescriptions are written for "off-label" uses for children because the drug doesn't have enough substantial evidence for FDA approval. The PNP should make sure to discuss the drug and this use with the family and document the decision-making process and their consent for this use. It is not enough to base a decision solely on what someone else has done. Unless the drug is experimental or has many serious adverse effects, close daily monitoring is not necessary. The PNP is not required to report off- label drug use to the FDA. 6. The primary care pediatric nurse practitioner prescribes a new medication for a child who develops a previously unknown adverse reaction. To report this, the nurse practitioner will : a. access the BPCA website. b. call the PREA hotline. c. log onto the FDA Medwatch website. d. use the AAP online PediaLink program. - Correct answer ANS: C The FDA Medwatch website is available for reporting of drug-related adverse effects, and all providers are encouraged to report these here. BPCA and PREA are legislative acts and do not have a hotline or website for adverse effects reporting. The AAP PediaLink program is a source for labeling changes of drugs. 7. The primary care pediatric nurse practitioner is treating a toddler who has a lower respiratory tract illness with a low-grade fever. The child is eating and taking fluids well and has normal oxygen saturations in the clinic. The nurse practitioner suspects that the child has a viral pneumonia and will : a. order an anti-viral medication and schedule a follow-up appointment. b. prescribe a broad-spectrum antibiotic until the lab results are received. c. teach the parents symptomatic care and order labs to help with the diagnosis. d. write a prescription for an antibiotic to be given if the child's condition worsens. - Correct answer ANS: C To decrease antibiotic overuse and resistance, the PNP should order an antibiotic only if laboratory data confirm a bacterial infection. This child is mildly ill and can be treated symptomatically. It is not necessary to treat with an anti-viral medication. A broad-spectrum antibiotic will only increase the risk of antibiotic resistance. Writing a prescription for the parents to fill if needed is not recommended; parents may give an antibiotic believing that it is indicated when it is not. 8. The primary care pediatric nurse practitioner is counseling an adolescent who was recently hospitalized for an asthma exacerbation and learns that the child usually forgets to use twice- daily inhaled corticosteroid medications that are supposed to be given at 0800 and 2000 each day. Which strategy may be useful in this case to improve adherence? P a g e 60 | 134 a. Ask the adolescent to identify two times each day that may work better. b. Consider having the school nurse supervise medication administration. c. Prescribing a daily oral corticosteroid medication instead. d. Suggest that the parent enforce the medication regimen each day. - Correct answer ANS: A When working with adolescents who take medication, it is important to allow the adolescent to have input into dosing schedules and what works for them. Having the school nurse supervise does not allow autonomy and creates continued dependency. Daily oral corticosteroids are not used for maintenance. The PNP should assist the family with transitioning the adolescent from parent to teen administration and not suggest that parents enforce medication rules. 1. A 10-month-old infant who is new to the clinic has chronic hepatitis B infection. What will the primary care pediatric nurse practitioner do to manage this infant's disease? a. Consult a pediatric infectious disease specialist. b. Prescribe interferon-alfa. c. Provide supportive care. d. Consider use of lamivudine. - Correct answer ANS: A A specialist in hepatitis B in children should be consulted for children with chronic hepatitis B infection because of the risk for developing hepatocellular carcinoma. Interferon-alfa and lamivudine are not used in infants. Supportive care only is not recommended. 2. The primary care pediatric nurse practitioner is performing an initial well child exam on a 3- year-old child recently adopted from Africa. The adoptive parent has a record of immunizations indicating that the child is fully vaccinated. What will the nurse practitioner do? a. Administer a booster dose of each vaccine to ensure immunity. b. Find out whether the vaccines were provided by reliable suppliers. c. Perform antibody titers and reimmunize the child. d. Record the vaccines in the child's electronic medical record. - Correct answer ANS: C Even though suppliers of vaccines worldwide produce vaccines that are of adequate quality, vaccine handling can be suspect. The PNP should perform titers and reimmunize if in doubt. If the child has not been adequately vaccinated, the PNP will need to administer each series based on catch-up dosing for age. If the child has been adequately immunized, boosters are not indicated. Performing titers is the best way to assess full immunity, since suppliers can be suspect. 3. A 3-year-old child who attends day care has had a fever, nausea, and vomiting several weeks prior and now has darkened urine and constipation along with hepatomegaly and right upper quadrant tenderness. What treatment is warranted for this child? a. HAV vaccine b. Immunoglobulin G c. Interferon-alfa d. Supportive care - Correct answer ANS: D The child has symptoms consistent with hepatitis A virus. HAV vaccine and IgG may be given within 2 weeks of exposure otherwise supportive care is indicated. Interferon-alfa is used for hepatitis B virus. - Correct answer 4. A 10-month-old infant has an erythematous, fluctuant, non-draining abscess on the right buttock after 10 days of treatment with amoxicillin for impetigo. What is the next step in managing this infant's care? a. Consultation with a pediatric infectious disease specialist b. Culture of any superficial open surface wounds P a g e 61 | 134 c. Empiric treatment with clindamycin d. Incision and drainage of the abscess with culture - Correct answer ANS: D Non-draining, fluctuant abscesses should be incised, drained, and cultured to determine the causative organism. Consultation with an infectious disease specialist is necessary for seriously ill children, those who are immunocompromised, or those who have an increased risk for myocarditis. Superficial wounds should not be cultured because of the chance of sample contamination. Empiric treatment may be considered for severe infection, but many mild abscesses may not need antibiotic therapy after I&D. 5. An unimmunized school-age child whose mother is in her first trimester of pregnancy is diagnosed with rubella after a local outbreak. What will the primary care pediatric nurse practitioner recommend? a. Assessment of maternal rubella titers b. Intravenous immunoglobulin for the child c. MMR vaccine for the mother and child d. Possible termination of the pregnancy - Correct answer ANS: A Reinfection or revaccination with rubella for pregnant women rarely results in congenital rubella syndrome, and these are not a reason for pregnancy termination. Maternal rubella antibody titers should be assessed. MMR vaccine is not given during pregnancy. IVIG is not indicated rubella rarely has serious sequelae in children. - Correct answer 6. A 2-month-old infant has a staccato cough and fever. Which aspect of the history is most important in determining the diagnosis? a. Day care attendance b. Immunization history c. Medication history d. Past medical history - Correct answer ANS: B A staccato cough may be present with pertussis, which is a vaccine-preventable disease. Careful assessment of immunization history is important when this is suspected. Day care attendance is an important aspect of determining exposure and may be considered, but it is not the most important part of the history in this case. Medication and past medical history are probably not relevant in this case since it is less likely that a 2-month-old infant has been taking medications or has a chronic or recurrent illness. 7. The parent of a 2-month-old infant is reluctant to have the baby vaccinated. What is an initial step in responding to these concerns? a. Inform the parent that all vaccines may be given without thimerosol. b. Providing Vaccine Information Statements for the parent to review. c. Question the parent's reasons for concern about immunizations. d. Remind the parent that the infant is exposed to thousands of germs each day. - Correct answer ANS: C PNPs should question and listen carefully to parents' concerns about vaccines. Once concerns are identified and understood, the PNP can address the issues. The presence of thimerosol in vaccines is just one concern and should be addressed if that is identified. Providing a Vaccine Information Statement (VIS) should be done as part of the discussion to provide information to the parent. Unless the parent expresses concerns that vaccines will overwhelm the child's immune system, it is not necessary to bring up this possibility . P a g e 62 | 134 16. The primary care pediatric nurse practitioner performs a well child examination on a 1- month-old. The infant was recently discharged from the neonatal intensive care unit after treatment with parenteral acyclovir for a neonatal herpetic infection and is currently taking oral acyclovir. What will the nurse practitioner do to manage this infant's care? a. Obtain regular absolute neutrophil counts. b. Perform routine skin cultures for herpes simplex virus. c. Reinforce the need to give acyclovir indefinitely. d. Stop the oral acyclovir at 2 months of age. - Correct answer ANS: A Infants treated with parenteral acyclovir for neonatal herpetic infections should remain on oral acyclovir for 6 months. The PNP should obtain regular ANC levels and temporarily discontinue acyclovir if neutropenia occurs until the neutrophil count recovers. New lesions should be cultured, but routine skin cultures are not indicated. Oral acyclovir suppressive therapy for 6 months after parenteral treatment of any classification of acute neonatal disease has been shown to reduce the recurrences of mucocutaneous lesions and improve neuro-developmental outcomes. 17. A child whose family has been camping in a region with endemic Lyme disease suffered several tick bites. The parents report removing the ticks but are not able to verify the type or the length of time the ticks were attached. The child is asymptomatic. What is the best course of action? a. Administer a prophylactic single dose of doxycycline. b. Perform serologic testing for IgG or IgM antibodies. c. Prescribe amoxicillin three times daily for 14 to 21 days. d. Teach the parents which signs and symptoms to report. - Correct answer ANS: D Prophylaxis should not be given if the type of tick or the timeline for attachment cannot be verified however, parents should be encouraged to report signs of Lyme disease if they occur. Prophylaxis is given if the tick is reliably identified as a nymph or adult Ixodes scapularis species. Serologic testing may be performed if symptoms occur. Amoxicillin tid for 2 to 3 weeks is indicated for early localized disease. - Correct answer 18. A child who is immunocompromised has a fever and a rash consisting of macules, papules, and pustules. What will the primary care pediatric nurse practitioner do? a. Administer varicella immune globulin (VariZIG). b. Hospitalize the child for intravenous acyclovir. c. Order intravenous immunoglobulin as an outpatient. d. Prescribe oral acyclovir for the duration of the illness. - Correct answer ANS: B The description of the rash the immunocompromised child has been exposed to is that of varicella. Intravenous acyclovir should be given to immunocompromised individuals. Immune globulin is not effective after the disease has progressed. Oral acyclovir is expensive and not routinely recommended for most children. 19. A preschool-age child is brought to clinic for evaluation of a rash. The primary care pediatric nurse practitioner notes an intense red eruption on the child's cheeks and circumoral pallor. What will the nurse practitioner tell the parents about this rash? a. This rash may be a prodromal sign of rubella or roseola. b. The child will need immunization boosters to prevent serious disease. c. This is a benign rash with no known serious complications. d. Expect a lacy, maculopapular rash to develop on the trunk and extremities. - Correct answer ANS: D This "slapped cheek" rash is consistent with fifth disease, or erythema infectiosum, and will be followed P a g e 65 | 134 by a lacy, maculopapular all-over rash. It is not a prodrome of rubella or roseola, and immunizations are not indicated. Although it is mostly benign, there can be serious sequelae, especially for pregnant women. 20. The primary care pediatric nurse practitioner is examining a 2-month-old infant with fever and cough. A WBC is 14,000/mm3 and a chest radiograph is normal. The infant is nursing well and having normal stools. What would be an appropriate next step? a. Admitting the infant to the hospital for LP and IV antibiotics b. Obtaining a blood culture, erythrocyte sedimentation rate, and C-reactive protein c. Performing a catheterized urinalysis to screen for leukocytes and nitrites d. Prescribing empiric, broad-spectrum antibiotics with close follow-up - Correct answer ANS: C A catheterized urinalysis to rule out UTI is appropriate to help determine the cause of infection. The infant has a reassuring WBC and chest X-ray, so it is not necessary to admit to the hospital or to perform blood cultures. Antibiotics are indicated only if bacterial infection is suspected. 21. A 2-month-old infant will receive initial immunizations, and the parent asks about giving medications to increase the infant's comfort and minimize fever. What will the primary care pediatric nurse practitioner recommend? a. Administering ibuprofen or acetaminophen as needed b. Avoiding antipyretics if possible to attain better immunity c. Giving ibuprofen and acetaminophen only after the vaccines d. Pre-treating the infant with both ibuprofen and acetaminophen - Correct answer ANS: A Although some studies have detected lower antibody responses in infants and children who were given antipyretics before or after routine vaccinations, the lowered response did not influence persistent immunological memory and did not cause a decrease in vaccine protection. Parents may administer acetaminophen or ibuprofen before or after vaccines as needed. There is no evidence that a combination of acetaminophen and ibuprofen is more effective. 22. According to recent research, which populations may have higher rates of under- immunization than others? a. Those with higher rates of Asians b. Those with higher rates of graduate degrees c. Those with lower rates of poverty d. Those with lower rates of primary providers - Correct answer ANS: B Rates of under-immunization were increased in populations with an increased percentage of graduate degrees in a study of geographic clusters of under-immunized communities in northern California. Populations having a higher percentage of Asians have increased immunization rates. Higher levels of poverty are associated with under-immunization. The study did not look at the effect of the number of primary providers. 23. The primary care pediatric nurse practitioner is reviewing medical records for a newborn that is new to the clinic. The toddler's mother was found to be HIV positive during her pregnancy with this child and received antiretroviral therapy during pregnancy. The child was born by cesarean section, begun on anti-retroviral prophylaxis, and did not breastfeed. What is the correct management for this child? a. Consult with a pediatric HIV specialist. b. Discontinue cART after 4 weeks of age. c. Obtain a CD4+ cell count and HIV RNA levels. d. Reinforce the need to give cART for life. - Correct answer ANS: A P a g e 66 | 134 PNPs may manage infants exposed in utero to HIV but should do so in consultation with a pediatric HIV specialist. cART should be given for 6 weeks. Lab work is ordered according to protocol at the direction of the specialist. Many children who are treated according to the protocol do not become HIV positive. 24. A child is brought to the clinic with a fever, headache, malaise, and a red, annular macule surrounded by an area of clearing and a larger, erythematous annular ring. The child complains of itching at the site. What will the primary care pediatric nurse practitioner do to determine the diagnosis? a. Ask about recent tick bites b. Obtain a skin culture c. Order blood cultures d. Perform serologic testing - Correct answer ANS: A The presence of an erythema migrans rash with a positive history is diagnostic for Lyme disease, and no further testing is necessary. Because Borrelia burgdorferi is transmitted to humans through ticks, asking about recent tick bites is paramount to making this diagnosis. Skin and blood cultures are not indicated. Serology testing for IgG and IgM antibodies may be performed if the child is symptomatic without the characteristic EM rash. 25. A 9-month-old infant has had a fever of 103°F for 2 days and now has a diffuse, maculopapular rash that blanches on pressure. The infant's immunizations are up-to-date. What will the primary care pediatric nurse practitioner do? a. Administer immunoglobulin G to prevent fulminant illness. b. Perform serologic testing for human herpes virus -6 and human herpes virus -7. c. Reassure the parent that this is a mild, self-limiting disease. d. Recommend avoiding contact with pregnant women. - Correct answer ANS: C The infant has symptoms consistent with roseola infantum, which is a benign, self-limiting disease. It is not necessary to administer IgG or perform serologic testing or to avoid contact with pregnant women. 26. The parent of an infant asks why some vaccines, such as MMR, are not given along with the other series of immunizations at 2, 4, and 6 months of age. What will the primary care pediatric nurse practitioner tell this parent? a. Febrile seizures are more likely in younger infants with some vaccines. b. Maternal antibodies neutralize some vaccines and are delayed until 12 months. c. The risk of adverse effects is lower for some vaccines after the first year. d. Too many vaccines at once can overwhelm the infant's immune system. - Correct answer ANS: B Maternal antibodies may neutralize certain vaccines, so some are delayed until the child is 1 year old. Febrile seizures and adverse reactions are not more likely in younger infants. There is no evidence that a large number of vaccines can overwhelm the infant's immune system. 27. A school-age child has fever of 104°F, sore throat, vomiting and malaise. The primary care pediatric nurse practitioner observes that the tonsils, oropharynx, and palate are erythematous and covered with exudate - Correct answer the tongue is coated and red - Correct answer and there is a red, sandpaper-like rash on the child's neck, trunk, and extremities. A rapid strep test is positive. What will the nurse practitioner do to manage this child's illness? a. Administer intramuscular ceftriaxone. P a g e 67 | 134 using the SABA. A treatment of 4 puffs of a SABA in clinic results in marked improvement in the child's status. What will the nurse practitioner do? a. a. Have the parent administer all of the child's medications. b. b. Increase the ICS medication to a high-dose preparation. c. c. Reinforce teaching about the importance of using the SABA. d. d. Teach the child and parent how to use home PEF monitoring. - Correct answer ANS: D Home PEF monitoring is useful for children to identify when symptoms are worsening. This child does not appear to notice the presence of airway tightness or wheezing and so might benefit from PEF monitoring to know when to use the SABA. School-age children should be learning how to manage their chronic disease, so having the parent administer all medications is not the best choice, especially since use of the SABA is still dependent on the child's report of symptoms. Since the child responded well to administration of the SABA, increasing the dose of ICS should not be done unless better management is not effective. Reinforcing the teaching is part of the plan but, unless the child is aware of symptoms, may not occur. 1. 4. A child has a fever and arthralgia. The primary care pediatric nurse practitioner learns that the child had a sore throat 3 weeks prior and auscultates a murmur in the clinic. Which test will the nurse practitioner order? a. a. Anti-DNase B test b. b. ASO titer c. c. Rapid strep test d. d. Throat culture - Correct answer ANS: B This child has symptoms and a history consistent with ARF. The ASO titer peaks in 3 to 6 weeks and will confirm a recent strep infection. The anti-DNase B test will also confirm a recent strep infection, but this doesn't peak until 6 to 8 weeks after the initial infection. A rapid strep test and throat culture do not differentiate the carrier state from a true infection. 1. 5. The primary care pediatric nurse practitioner is prescribing ibuprofen for a 25 kg child with JIA who has oligoarthitis. If the child will take 4 doses per day, what is the maximum amount the child will receive per dose? a. a. 200 mg b. b. 250 mg c. c. 400 mg d. d. 450 mg - Correct answer ANS: B The maximum dose is 40 mg/kg/day divided into 3 to 4 doses. 25 kg × 40 mg = 1000/4 = 250 mg. 1. 6. A school-age child who uses a SABA and an inhaled corticosteroid medication is seen in the clinic for an acute asthma exacerbation. After 4 puffs of an inhaled short-acting B2-agonist (SABA) every 20 minutes for three treatments, spirometry testing shows an FEV1 of 60% of the child's personal best. What will the primary care pediatric nurse practitioner do next? a. a. Administer an oral corticosteroid and repeat the three treatments of the inhaled SABA. b. b. Admit the child to the hospital for every 2 hour inhaled SABA and intravenous steroids. c. c. Give the child 2 mg/kg of an oral corticosteroid and have the child taken to the emergency department. d. d. Order an oral corticosteroid, continue the SABA every 3 to 4 hours, and follow closely. - Correct answer ANS: D Children with an incomplete response (FEV1 between 40% and 69% of personal best) should be given oral steroids and instructed to continue the SABA every 3 to 4 hours with close follow- up. Hospitalization is not necessary unless severe distress occurs. An FEV1 less than 40% after treatment indicates a need to be seen in the ED. 1. 7. An adolescent who has asthma and severe perennial allergies has poor asthma control in spite of appropriate use of a SABA and a daily high-dose inhaled corticosteroid. What will the primary care pediatric nurse practitioner do next to manage this child's asthma? a. a. Consider daily oral corticosteroid administration. b. b. Order an anticholinergic medication in conjunction with the current regimen. c. c. Prescribe a LABA/inhaled corticosteroid combination medication. d. d. Refer to a pulmonologist for omalizumab therapy. - Correct answer ANS: D Children older than 12 years who have moderate to severe allergy-related asthma and who P a g e 70 | 134 react to perennial allergens may benefit from omalizumab as a second-line treatment when symptoms are not controlled by ICSs. The PNP should refer children to a pulmonologist for such treatment. Daily oral corticosteroid medications are not recommended because of the adverse effects caused by prolonged use of this route. Anticholinergic medications are generally used for acute exacerbations during in-patient stays or in the ED. A LABA/ICS combination will not produce different results. 1. 8. A 4-month-old infant has a history of reddened, dry, itchy skin. The primary care pediatric nurse practitioner notes fine papules on the extensor aspect of the infant's arms, anterior thighs, and lateral aspects of the cheeks. What is the initial treatment? a. a. Moisturizers b. b. Oral antihistamines c. c. Topical corticosteroids d. d. Wet wrap therapy - Correct answer ANS: A Moisturization is the first-line therapy to interrupt the itch-scratch-itch cycle. Oral antihistamines are used mostly to allow sleep during nighttime pruritus. Topical corticosteroids are used if moisturization is not effective. Wet wrap therapy is used to treat flares with recalcitrant disease. 1. 9. An 8-year-old child is diagnosed with systemic lupus erythematosus (SLE), and the child's parent asks if there is a cure. What will the primary care pediatric nurse practitioner tell the parent? a. a. Complete remission occurs in some children at the age of puberty. b. b. Periods of remission may occur but there is no permanent cure. c. c. SLE can be cured with effective medication and treatment. d. d. The disease is always progressive with no cure and no remissions. - Correct answer ANS: B Periods of remission do occur in some children with SLE for unknown reasons, but there is no permanent remission or cure. For some children with Juvenile Idiopathic Arthritis (JIA), complete remission occurs at puberty. 1. 10. The primary care pediatric nurse practitioner is examining a school-age child who has had several hospitalizations for bronchitis and wheezing. The parent reports that the child has several coughing episodes associated with chest tightness each week and gets relief with an albuterol metered-dose inhaler. What will the nurse practitioner order? a. a. Allergy testing b. b. Chest radiography c. c. Spirometry testing d. d. Sweat chloride test - Correct answer ANS: C Spirometry testing is the gold standard for diagnosing asthma and is then used on a regular basis to monitor, evaluate, and manage asthma. Allergy testing should be considered but is not diagnostic of asthma. Chest radiography should not be routine. A sweat chloride test is used based on history. 1. 11. The primary care pediatric nurse practitioner examines a child who has had stiffness and warmth in the right knee and left ankle for 7 or 8 months but no back pain. The nurse practitioner will refer the child to a rheumatology specialist to evaluate for a. a. enthesitis-related JIA. b. b. oligoarticular JIA. c. c. polyarticular JIA. d. d. systemic JIA. - Correct answer ANS: B Oligoarticular JIA is characterized by mild, painless asymmetric joint involvement without systemic symptoms. Enthesitis-related JIA involves arthritis of the lower limbs, especially the hips, intertarsal joints, and sacroiliac joints, with swelling, tenderness, and warmth. Polyarticular JIA involves 5 or more joints. Systemic JIA presents with systemic symptoms, such as fever. 1. 12. A child who has been diagnosed with asthma for several years has been using a short-acting B2-agonist (SABA) to control symptoms. The primary care pediatric nurse practitioner learns that the child has recently begun using the SABA two or three times each week to treat wheezing and shortness of breath. The child currently has clear breath sounds and an FEV1 of 75% of personal best. What will the nurse practitioner do next? a. a. Add a daily inhaled corticosteroid. b. b. Administer 3 SABA treatments. c. c. Continue the current treatment. d. d. Order an oral corticosteroid. - Correct answer ANS: A The child is showing a need to step up treatment based on the frequency of symptoms, greater than twice each week. The PNP should order an inhaled corticosteroid maintenance medication to control symptoms and reduce the need for a SABA. The child is not having an acute P a g e 71 | 134 exacerbation, so does not need 3 SABA treatments. Oral corticosteroids are given for moderate obstruction, <70%. 1. 13. The primary care pediatric nurse practitioner is evaluating an 11-month-old infant who has had three viral respiratory illnesses causing bronchiolitis. The child's parents both have seasonal allergies and ask whether the infant may have asthma. What will the nurse practitioner tell the parents? a. a. "Although it is likely, based on family history, it is too soon to tell." b. b. "There is little reason to suspect that your infant has asthma." c. c. "With your infant's history of bronchiolitis, asthma is very likely." d. d. "Your infant has definitive symptoms consistent with a diagnosis of asthma." - Correct answer ANS: A A genetic predisposition for the development of an IgE-mediated response to aeroallergens is the strongest identifiable predisposing risk factor for asthma, but asthma is rarely diagnosed before age 12 months due to the high rate of viral-induced bronchiolitis. The PNP should be cautious about diagnosing asthma until wheezing without an association to viral illnesses occurs. This infant has clear risk factors for asthma however, bronchiolitis is not a known risk factor. - Correct answer 1. 14. An 8-year-old boy has a recent history of an upper respiratory infection and comes to the clinic with a maculopapular rash on his lower extremities and swelling and tenderness in both ankles. The pediatric nurse practitioner performs a UA, which shows proteinuria and hematuria and diagnoses HSP. What ongoing evaluation will the nurse practitioner perform during the course of this disease? a. a. ANA titers b. b. Blood pressure measurement c. c. Chest radiographs d. d. Liver function studies - Correct answer ANS: B Hypertension is a serious risk of HSP, so repeated BP measurement is indicated. ANA titers are not measured with HSP. Chest radiographs are performed only if indicated. LFTs are not indicated the predominant risk is to the kidneys. - Correct answer 1. 15. A 12-year-old child is brought to the clinic with joint pain, a 3-week history of low- grade fever, and a facial rash. The primary care pediatric nurse practitioner palpates an enlarged liver 2 cm below the subcostal margin along with diffuse lymphadenopathy. An ANA test is positive. Which test may be ordered to confirm a diagnosis of SLE? a. a. Anti-double-strand DNA antibodies b. b. Anti-La antibodies c. c. Anti-Ro antibodies d. d. Anti-Sm antibodies - Correct answer ANS: A Anti-double-strand DNA antibodies are present in most people with SLE and are generally exclusively seen in cases of SLE and not other diseases. Anti-SM antibodies are diagnostic of SLE but are only seen in 30% of patients with SLE. 1. 16. A 10-year-old child has a 1-week history of fever of 104°C that is unresponsive to antipyretics. The primary care pediatric nurse practitioner examines the child and notes bilateral conjunctival injection and a polymorphous exanthema, with no other symptoms. Lab tests show elevated ESR, CRP, and platelets. Cultures are all negative. What will the nurse practitioner do? a. a. Begin treatment with intravenous methyl prednisone. b. b. Consider IVIG therapy if symptoms persist one more week. c. c. Order a baseline echocardiogram today and another in 2 weeks. d. d. Reassure the child's parents that this is a self-limiting disorder. - Correct answer ANS: C An echocardiogram should be obtained as soon as the diagnosis of Kawasaki disease (KD) is established, as a baseline study, with subsequent studies in 2 weeks and in 6 to 8 weeks. This child has fever and only two other symptoms, which may be consistent with atypical KD. Atypical KD is more common in very young children and in children over 9 years of age, and coronary artery involvement is found more frequently in children with atypical KD. Methyl prednisone is given for children with IVIG-resistant disease. IVIG should be begun ideally in the first 10 days of the illness. Although KD is a self-limiting disorder, the risk of coronary artery involvement is high, so this must be evaluated and treated. P a g e 72 | 134 a. bone marrow biopsy. b. corticosteroids and IVIG. c. hemoglobin electrophoresis. d. immunoglobulin testing. - Correct answer ANS: A This child has symptoms and initial lab tests consistent with leukemia and should be referred to a pediatric hematologist-oncologist for a bone marrow biopsy for a definitive diagnosis. Corticosteroids and IVIG are given for severe ITP. Hgb electrophoresis is used to diagnose SCA. Immunoglobulins are evaluated when immune deficiency syndromes are suspected. 5. The pediatric nurse practitioner provides primary care for a 30-month-old child who has sickle cell anemia who has had one dose of 23-valent pneumococcal vaccine. Which is an appropriate action for health maintenance in this child? a. Administer an initial meningococcal vaccine. b. Begin folic acid dietary supplementation. c. Decrease the dose of penicillin V prophylaxis. d. Give a second dose of 23-valent pneumococcal vaccine. - Correct answer ANS: A Invasive bacterial infection is the leading cause of death in young children with SCA. Meningococcal vaccine should be given initially for all children over the age of 2 years and a booster dose given every 5 years after that. Folic acid supplementation is often used for adults but not for children unless there is a documented deficiency. Penicillin V prophylaxis is started at 2 months of age, with the dose increased at age 3 years. The 23-valent pneumococcal polysaccharide second dose is given 5 years after the first. 6. The primary care pediatric nurse practitioner reviews hematology reports on a child with beta- thalassemia minor and notes an Hgb level of 8 g/dL. What will the nurse practitioner do? a. Evaluate serum ferritin. b. Order Hgb electrophoresis. c. Prescribe supplemental iron. d. Refer for RBC transfusions. - Correct answer ANS: A Children with beta-thalassemia minor may have low hemoglobin without iron deficiency so, before prescribing iron, the PNP should measure serum iron levels or serum ferritin. Hgb electrophoresis is indicated in a child whose diagnosis is unknown to diagnose this disorder. Supplemental iron should only be ordered when there is documented iron deficiency. RBC transfusions are controversial and used only for more severe iron deficiency. 7. A school-age child comes to the clinic for evaluation of excessive bruising. The primary care pediatric nurse practitioner notes a history of an upper respiratory infection 2 weeks prior. The physical exam is negative for hepatosplenomegaly and lymphadenopathy. Blood work reveals a platelet count of 60,000/mm3 with normal PT and aPTT. How will the nurse practitioner manage this child's condition? a. Admit to the hospital for IVIG therapy. b. Begin a short course of corticosteroid therapy. c. Refer to a pediatric hematologist. d. Teach to avoid NSAIDs and contact sports. - Correct answer ANS: D This child has symptoms, a history, and lab work that indicate idiopathic thrombocytopenic purpura. Since platelets are greater than 20,000/mm3, management without specific therapy may be done on an outpatient basis by teaching the family to avoid things that contribute to bleeding. IVIG therapy is used for children with active, severe bleeding. Corticosteroids are given for platelet counts less than 20,000/mm3. Referral to a hematologist is necessary for more severe cases. P a g e 75 | 134 8. A toddler who presents with anemia and reticulocytopenia has a history of a gradual decrease in energy and increase in pallor beginning after a recent viral infection. How will the primary care pediatric nurse practitioner treat this child? a. Closely observe the child's symptoms and lab values. b. Consult with a pediatric hematologist. c. Prescribe supplemental iron for 4 to 6 months. d. Refer for transfusions to correct the anemia. - Correct answer ANS: A This child has symptoms and a history consistent with transient erythroblastopenia of childhood (TEC), which is usually self-limited. The PNP should monitor the child closely without treatment unless the anemia gets worse. Any of the other options may be necessary if the child's condition worsens. 9. The primary care pediatric nurse practitioner is managing care for a child diagnosed with iron- deficiency anemia who had an initial hemoglobin of 8.8 g/dL and hematocrit of 32% who has been receiving ferrous sulfate as 3 mg/kg/day of elemental iron for 4 weeks. The child's current lab work reveals elevations in Hgb/Hct and reticulocytes with a hemoglobin of 10.5 g/dL and a hematocrit of 36%. What is the next step in management of this patient? a. Continue the current dose of ferrous sulfate and recheck labs in 1 to 2 months. b. Discontinue the supplemental iron and encourage an iron-enriched diet. c. Increase the ferrous sulfate dose to 4 to 6 mg/kg/day of elemental iron. d. Refer the child to a pediatric hematologist to further evaluate the anemia. - Correct answer ANS: A This child has mild to moderate anemia and is showing a good response to the current dose of iron. Ferrous sulfate should be continued for at least 2 to 3 months to normalize hemoglobin, and then continue for 2 to 4 months to replace depleted iron stores. There is no need to increase the dose, since the child is responding appropriately to the current dose. Children with hemoglobin levels less than 4 g/dL should be referred. 10. The primary care pediatric nurse practitioner is performing a well child examination on a school-age child who has a history of cancer treated with cranial irradiation. What will the nurse practitioner monitor in this child? a. Cardiomyopathy and arrhythmias b. Leukoencephalopathy c. Obesity and gonadal dysfunction d. Peripheral neuropathy and hearing loss - Correct answer ANS: B Leukoencephalopathy is a late effect of cancer treatment associated with cranial irradiation. Cardiomyopathy and arrhythmias are related to anthracycline use. Obesity and gonadal dysfunction result from neuroendocrine effects of chemotherapeutic agents. Peripheral neuropathy and hearing loss occur after cisplatin use. 11. A complete blood count on a 12-month-old infant reveals microcytic, hypochromic anemia with a hemoglobin of 9.5 g/dL. The infant has mild pallor with no hepatosplenomegaly. The primary care pediatric nurse practitioner suspects a. hereditary spherocytosis. b. iron-deficiency anemia. c. lead intoxication. d. sickle-cell anemia. - Correct answer ANS: B Iron-deficiency anemia is the most common type of anemia in infants and children, accounting for approximately 90% of cases. It is characterized by decreased hemoglobin, with microcytic, hypochromic P a g e 76 | 134 RBCs. Hereditary spherocytosis is characterized by pallor and jaundice with splenomegaly. Lead intoxication is accompanied by neurobehavioral problems. Sickle-cell anemia involves the presence of HgbS. 12. The primary care pediatric nurse practitioner performs a well baby examination on a 4- month-old infant who is exclusively breastfed and whose mother plans to introduce only small amounts of fruits and vegetables in addition to breastfeeding. To ensure that the infant gets adequate amounts of iron, what will the nurse practitioner recommend? a. Elemental iron supplementation of 1 mg/kg/day until cereals are added b. Elemental iron supplementation of 3 mg/kg/day for the duration of breastfeeding c. Monitoring the infant's hemoglobin and hematocrit at every well-baby checkup d. Offering iron-fortified formula to ensure adequate iron intake - Correct answer ANS: A Infants who are exclusively breastfeeding or who receive more than half of their diet from breast milk should be given 1 mg/kg/day of supplemental iron until iron-containing foods are added to the diet. It is not necessary to monitor Hgb/Hct regularly unless the child has symptoms. Formula is not necessary for breastfeeding infants. 13. The primary care pediatric nurse practitioner reviews a child's complete blood count with differential white blood cell values and recognizes a "left shift" because of a. a decreased eosinophil count. b. a decreased lymphocyte count. c. an elevated monocyte count. d. an elevated neutrophil count. - Correct answer ANS: D A left shift occurs when there is an increase in the number of circulating immature neutrophils and indicates a bacterial infection or an inflammatory disorder. Eosinophils are associated with an antigen- antibody response and are elevated with exposure to allergens, inflammation of skin, or parasites. Lymphocytes are non-granulocytes that are elevated with viral infections. Monocytes are non- granulocytes and are elevated in infections and inflammation and some leukemias, elevations of non- granulocytes are referred to as a "right shift." 1. A 10-month-old infant who is new to the clinic has chronic hepatitis B infection. What will the primary care pediatric nurse practitioner do to manage this infant's disease? a. Consult a pediatric infectious disease specialist. b. Prescribe interferon-alfa. c. Provide supportive care. d. Consider use of lamivudine. - Correct answer ANS: A A specialist in hepatitis B in children should be consulted for children with chronic hepatitis B infection because of the risk for developing hepatocellular carcinoma. Interferon-alfa and lamivudine are not used in infants. Supportive care only is not recommended. 2. The primary care pediatric nurse practitioner is performing an initial well child exam on a 3- year-old child recently adopted from Africa. The adoptive parent has a record of immunizations indicating that the child is fully vaccinated. What will the nurse practitioner do? a. Administer a booster dose of each vaccine to ensure immunity. b. Find out whether the vaccines were provided by reliable suppliers. c. Perform antibody titers and reimmunize the child. d. Record the vaccines in the child's electronic medical record. - Correct answer ANS: C Even though suppliers of vaccines worldwide produce vaccines that are of adequate quality, vaccine P a g e 77 | 134 c. There is some risk of CNS disorders associated with the hepatitis B vaccine. d. Vaccines containing thimerosol are linked to pervasive developmental disorders. - Correct answer ANS: B The IOM report found that febrile seizures and measles inclusion body encephalitis can occur in immunocompromised children. The IOM found no substantiated evidence that multiple vaccines trigger type 1 diabetes, hepatitis B vaccine is associated with increased risk of CNS disorders, or thimerosol- containing vaccines are linked to pervasive developmental disorders. 11. An adolescent female who is sexually active and who has not had the HPV vaccine asks if she may have it. What will the primary care pediatric nurse practitioner tell her? a. Getting the vaccine now will still protect her from HPV oncogenic types even if already exposed b. Receiving the HPV vaccine series will replace the need for regular cervical cancer screening c. She will need to have Papanicolaou and pregnancy screening prior to receiving the vaccine d. The vaccine will not protect her from any HPV oncogenic types acquired previously - Correct answer ANS: D There is no protection for HPV oncogenic types acquired prior to the vaccine. The vaccine does not eliminate the need for cervical cancer screening. It is not necessary to perform PAP or pregnancy testing prior to vaccine administration. 12. A child with a history of a pustular rash at the site of a cat scratch on one arm now has warm, tender, swollen axillary lymph nodes on the affected side. The primary care pediatric nurse practitioner notes induration and erythema of these nodes. What will the nurse practitioner do? a. Obtain a complete blood count and C-reactive protein. b. Order an immunofluorescent assay (IFA) for serum antibodies. c. Perform a needle aspiration of the affected lymph nodes. d. Prescribe a 5-day course of azithromycin. - Correct answer ANS: B IFA shows a good correlation with cat-scratch fever disease and is useful for a more definitive diagnosis. A complete blood count and C-reactive protein are non-specific indicators of disease. Needle aspiration is only necessary to determine whether local lymph nodes are infected. Antibiotics are not given unless nodes are infected. 13. An 18-month-old child has bronchopulmonary dysplasia. To help prevent pneumococcal disease, which vaccine will be ordered? a. PCV7 b. PCV13 c. PCV23 d. PCV33 - Correct answer ANS: B PCV13 is recommended for all children under age 5 years. PCV7 was replaced by PCV13 in 2010. PCV23 is used in children over age 2 years who are at higher risk of pneumococcal disease. this child will be eligible for the PCV23 at age 2. children younger than 2 years have shown poor immunogenicity to PCV23. PCV33 does not exist. 14. A child whose parents have refused vaccines has been exposed to chickenpox, and the parents ask whether the child may attend day care. What will the primary care pediatric nurse practitioner tell them? a. The child may attend day care as long as no rash is present even with mild fever or other symptoms. b. The child should remain home and receive oral acyclovir for 5 days to prevent onset of symptoms. P a g e 80 | 134 c. The child should stay home until the 21-day incubation period has passed even if symptom free. d. The child should stay home if any symptoms occur and may return in 1 week if no rash develops. - Correct answer ANS: D Exposed children can attend school for about a week but should be kept out of school if any symptoms develop for 1 week and may return if no rash develops. Any symptomatic child should be kept home for 1 week. Oral acyclovir is not effective in preventing disease onset. It is not necessary to remain home for the duration of the incubation period. 15. The primary care pediatric nurse practitioner reviews the immunization records of an 18- month-old child and notes that the child received an MMR immunization 2 days prior to the first birthday. What will the nurse practitioner do? a. Administer a reduced dose of MMR to ensure adequate immunity. b. Obtain mumps, measles, and rubella titers to determine immunity. c. Recommend the next dose of MMR vaccine at 4 to 5 years of age. d. Repeat the MMR vaccine since the first dose was given too soon - Correct answer ANS: C Vaccine doses may be given 4 days prior to or later than minimum intervals or ages to provide schedule flexibility. The next dose will be given at age 4 to 5, so this child may remain on schedule. It is never recommended to give reduced doses for any reason. reimmunization if in doubt is not harmful. Titers are not indicated unless the vaccine quality or storage is in doubt. 16. The primary care pediatric nurse practitioner performs a well child examination on a 1- month-old. The infant was recently discharged from the neonatal intensive care unit after treatment with parenteral acyclovir for a neonatal herpetic infection and is currently taking oral acyclovir. What will the nurse practitioner do to manage this infant's care? a. Obtain regular absolute neutrophil counts. b. Perform routine skin cultures for herpes simplex virus. c. Reinforce the need to give acyclovir indefinitely. d. Stop the oral acyclovir at 2 months of age. - Correct answer ANS: A Infants treated with parenteral acyclovir for neonatal herpetic infections should remain on oral acyclovir for 6 months. The PNP should obtain regular ANC levels and temporarily discontinue acyclovir if neutropenia occurs until the neutrophil count recovers. New lesions should be cultured, but routine skin cultures are not indicated. Oral acyclovir suppressive therapy for 6 months after parenteral treatment of any classification of acute neonatal disease has been shown to reduce the recurrences of mucocutaneous lesions and improve neuro-developmental outcomes. 17. A child whose family has been camping in a region with endemic Lyme disease suffered several tick bites. The parents report removing the ticks but are not able to verify the type or the length of time the ticks were attached. The child is asymptomatic. What is the best course of action? a. Administer a prophylactic single dose of doxycycline. b. Perform serologic testing for IgG or IgM antibodies. c. Prescribe amoxicillin three times daily for 14 to 21 days. d. Teach the parents which signs and symptoms to report. - Correct answer ANS: D Prophylaxis should not be given if the type of tick or the timeline for attachment cannot be verified. however, parents should be encouraged to report signs of Lyme disease if they occur. Prophylaxis is given if the tick is reliably identified as a nymph or adult Ixodes scapularis species. Serologic testing may be performed if symptoms occur. Amoxicillin tid for 2 to 3 weeks is indicated for early localized disease. P a g e 81 | 134 18. A child who is immunocompromised has a fever and a rash consisting of macules, papules, and pustules. What will the primary care pediatric nurse practitioner do? a. Administer varicella immune globulin (VariZIG). b. Hospitalize the child for intravenous acyclovir. c. Order intravenous immunoglobulin as an outpatient. d. Prescribe oral acyclovir for the duration of the illness. - Correct answer ANS: B The description of the rash the immunocompromised child has been exposed to is that of varicella. Intravenous acyclovir should be given to immunocompromised individuals. Immune globulin is not effective after the disease has progressed. Oral acyclovir is expensive and not routinely recommended for most children. 19. A preschool-age child is brought to clinic for evaluation of a rash. The primary care pediatric nurse practitioner notes an intense red eruption on the child's cheeks and circumoral pallor. What will the nurse practitioner tell the parents about this rash? a. This rash may be a prodromal sign of rubella or roseola. b. The child will need immunization boosters to prevent serious disease. c. This is a benign rash with no known serious complications. d. Expect a lacy, maculopapular rash to develop on the trunk and extremities. - Correct answer ANS: D This "slapped cheek" rash is consistent with fifth disease, or erythema infectiosum, and will be followed by a lacy, maculopapular all-over rash. It is not a prodrome of rubella or roseola, and immunizations are not indicated. Although it is mostly benign, there can be serious sequelae, especially for pregnant women. 20. The primary care pediatric nurse practitioner is examining a 2-month-old infant with fever and cough. A WBC is 14,000/mm3 and a chest radiograph is normal. The infant is nursing well and having normal stools. What would be an appropriate next step? a. Admitting the infant to the hospital for LP and IV antibiotics b. Obtaining a blood culture, erythrocyte sedimentation rate, and C-reactive protein c. Performing a catheterized urinalysis to screen for leukocytes and nitrites d. Prescribing empiric, broad-spectrum antibiotics with close follow-up - Correct answer ANS: C A catheterized urinalysis to rule out UTI is appropriate to help determine the cause of infection. The infant has a reassuring WBC and chest X-ray, so it is not necessary to admit to the hospital or to perform blood cultures. Antibiotics are indicated only if bacterial infection is suspected. 21. A 2-month-old infant will receive initial immunizations, and the parent asks about giving medications to increase the infant's comfort and minimize fever. What will the primary care pediatric nurse practitioner recommend? a. Administering ibuprofen or acetaminophen as needed b. Avoiding antipyretics if possible to attain better immunity c. Giving ibuprofen and acetaminophen only after the vaccines d. Pre-treating the infant with both ibuprofen and acetaminophen - Correct answer ANS: A Although some studies have detected lower antibody responses in infants and children who were given antipyretics before or after routine vaccinations, the lowered response did not influence persistent immunological memory and did not cause a decrease in vaccine protection. Parents may administer acetaminophen or ibuprofen before or after vaccines as needed. There is no evidence that a combination of acetaminophen and ibuprofen is more effective. 22. According to recent research, which populations may have higher rates of under- immunization than others? a. Those with higher rates of Asians P a g e 82 | 134 are unimmunized or under-immunized will contract the disease if exposed. 31. An 18-month-old child who developed upper respiratory symptoms 1 day prior is brought to the clinic with a high fever, chills, muscle pains, and a dry, hacking cough. A rapid influenza test is negative and a viral culture is pending. What will the primary care pediatric nurse practitioner do? a. Consider therapy with rimantadine. b. Hospitalize for supportive treatment. c. Prescribe oseltamivir and follow closely d. Wait for cultures to determine treatment. - Correct answer ANS: C A negative rapid viral culture should not be the determining factor when deciding on a clinical course of treatment when influenza is suspected. Children under age 2 years should be treated with antiviral medications. Rimantadine is not recommended unless susceptibility is reliable. cultures will not be confirmed for several days. It is not necessary to hospitalize unless the child has signs of respiratory distress or cardiac involvement. Antivirals should be initiated within 72 hours of onset of symptoms. 32. When reviewing a white blood cell (WBC) count, the primary care pediatric nurse practitioner suspects a viral infection when which WBC element is elevated? a. Bands b. Leukocytes c. Lymphocytes d. Neutrophils - Correct answer ANS: C Lymphocytes are usually elevated during viral infections. Bands and neutrophils are generally elevated with bacterial infections. Leukocytes comprise all WBCs and are usually, although not always, elevated during bacterial infections. 33. A 5-year-old child who received VariZIG after exposure to varicella while immunocompromised during chemotherapy is in the clinic 5 months after stopping chemotherapy for kindergarten vaccines. What will the primary care pediatric nurse practitioner order for this child? a. MMR and Tdap b. MMR, Varivax, Tdap c. Tdap only d. Varivax and Tdap - Correct answer ANS: B This child is eligible for all three vaccines. Varivax should be given 5 months after VariZIG, unless varicella disease occurred despite VariZIG administration. 34. A 7-year-old child whose immunizations are up-to-date has a fever, headache, stiff neck, and photophobia. What course of treatment is indicated? a. Empiric treatment with oral antibiotics or intramuscular ceftriaxone b. Hospitalization for diagnosis and treatment with antibiotics c. Immediate vaccination with meningococcal vaccine d. Outpatient lab work, including a CBC and blood and CSF cultures - Correct answer ANS: B Any child suspected of having meningococcal meningitis should be hospitalized immediately with IV antibiotics started pending cultures. Empiric treatment is not indicated. Vaccination is not helpful once the disease has started. Outpatient lab work is not indicated. 1. The parents of an 18-month-old child bring the child to the clinic after observing a brief seizure of less than 2 minutes in their child. In the clinic, the child has a temperature of 103.1°F, and the primary care pediatric nurse practitioner notes a left otitis media. The child is alert and responding normally. What will the nurse practitioner do? P a g e 85 | 134 a. Order a lumbar puncture, complete blood count, and urinalysis. b. Prescribe an antibiotic for the ear infection and reassure the parents. c. Refer to a pediatric neurologist for anticonvulsant and antipyretic prophylaxis. d. Send the child to the emergency department for EEG and possible MRI. - Correct answer ANS: B This child has symptoms of a simple febrile seizure with a focal site of infection and an otherwise normal exam. While this is very frightening to the family, the PNP should treat the infection and provide reassurance to the parents. Lumbar puncture may be performed in infants younger than 12 months. Prophylactic medications aren't indicated for febrile seizures. Antipyretics aren't useful, since most seizures occur when the temperature is either rising or falling. EEG and MRI are not indicated when focal neurological signs are not present. 2. A child who has sustained a head injury after falling on the playground is brought to the clinic. The parents report that the child cried immediately and was able to walk around after falling. The primary care pediatric nurse practitioner notes slight slurring of the child's speech and the child has vomited twice in the exam room. Which course of action is warranted? a. Admit the child to the hospital for a neurology consult. b. Observe the child in the clinic for several hours. c. Order a head CT and observe the child at home. d. Send the child home with instructions for follow-up. - Correct answer ANS: A Children with certain symptoms, such as vomiting or slurred speech after a head injury, should be admitted to the hospital for neurologic consultation. If the child had not exhibited these symptoms, any of the other options would be acceptable. 3. A female infant who was developing normally stops meeting developmental milestones at age 12 months and then begins losing previously acquired skills. What will the primary care pediatric nurse practitioner expect to tell the parents about this child's prognosis? a. Cognitive development will be normal but motor skills will be lost. b. Physical and speech therapy will help the infant regain lost skills. c. The child's intellectual development will not progress further. d. This is a temporary condition with full recovery expected. - Correct answer ANS: C This child has symptoms of Rett syndrome, which affects females more than males and is characterized by a plateau of development with eventual loss of milestones. Intellectual development remains at the level of plateau. Physical therapy, occupational therapy, and speech therapy help to preserve functional abilities but do not improve skills. The condition is progressive, with variable life expectancy. 4. To evaluate brain tissue disorders in infants, which test is useful? a. Computerized tomography b. Head radiographs c. Magnetic resonance imaging d. Ultrasonography - Correct answer ANS: D Ultrasonography is used to evaluate brain tissue in infants. CT scans expose patients to high levels of radiation, so they are not used unless indicated. Radiographs have relatively diagnostic value for the neurologic system. Magnetic resonance imaging is useful but is expensive and usually requires sedation. 5. A child who has had a single non-febrile seizure has a normal neurologic exam. Which diagnostic test is indicated? a. Computerized tomography (CT) b. Electroencephalogram (EEG) c. Magnetic resonance imaging (MRI) P a g e 86 | 134 d. Polysomnography - Correct answer ANS: B An EEG is standard for all children after a first non-febrile seizure. CT is not routinely used because of radiation exposure. MRI is used if cognitive changes or postictal focal dysfunction persists, if the seizure lasts longer than 15 minutes, if the child is younger than 6 months of age, and if any new onset of focal neurologic deficit has occurred. Polysomnography is used to assess nocturnal seizures. 6. During a well baby exam on a 9-month-old infant, the parent reports that the baby always uses the left hand to pick up objects and asks if the baby will be left-handed. What will the primary care pediatric nurse practitioner do? a. Explain that it is too soon to tell which hand the infant will prefer later. b. Perform a careful assessment of fine and gross motor skills. c. Teach the parent to encourage the infant to use both hands. d. Tell the parent that a hand preference usually develops between 6 and 12 months. - Correct answer ANS: B Hand preference before 1 year of age is usually suspect for cerebral palsy and may indicate a lack of motor skills in the other hand. The PNP should perform a careful assessment of fine and gross motor skills. Infants should not exhibit a hand preference until after 1 year of age, so the correct response is to assess further. 7. Because of their inability to ambulate, children with cerebral palsy should be evaluated for which nutrients? a. Calcium and vitamin D b. Fat-soluble vitamins c. Iron and zinc d. Sodium and potassium - Correct answer ANS: A Children who do not place weight on their bones are at risk for osteopenia and should have vitamin D and calcium levels monitored and supplemented if indicated. 8. A 14-year-old child has a headache, unilateral weakness, and blurred vision preceded by fever and nausea. The child's parent reports a similar episode several months prior. The primary care pediatric nurse practitioner will consult with a pediatric neurologist to order a. a lumbar puncture. b. an electroencephalogram (EEG). c. neuroimaging with magnetic resonance imaging (MRI). d. positron emission tomography (PET) scan. - Correct answer ANS: C Children who have MS exhibit the symptoms described above and are usually diagnosed with a gadolinium enhanced MRI. Lumbar puncture may be performed later to identify oligoclonal bands. An EEG is used to diagnose seizure activity. PET scans are used to detect tumors. 9. The pediatric nurse practitioner provides primary care for a 5-year-old child who has cerebral palsy who exhibits athetosis and poor weight gain in spite of receiving high-calorie formula to supplement intake. The child has had several episodes of pneumonia in the past year. Which specialty consultation is a priority for this child? a. Feeding clinic to manage caloric intake b. Neurology to assess medication needs c. Pulmonology for possible tracheotomy d. Surgery for possible fundoplication and gastrostomy - Correct answer ANS: D Children with CP who have athetosis often have increased calorie needs up to 50% to 100% higher than others. This child is unable to gain adequate weight in spite of receiving extra calories. The child also has P a g e 87 | 134 infant who has a history of prematurity at 28 weeks' gestation. The infant was treated for retinopathy of prematurity (ROP) and all symptoms have resolved. When will the infant need an ophthalmologic exam? a. At 12 months of age b. At 24 months of age c. At 48 months of age d. At 60 months of age - Correct answer ANS: A Children who have a history of ROP requiring treatment, even if ROP has completely resolved, will need yearly ophthalmologic follow-up. Less frequent follow-up is required for children with ROP who did not require treatment. 3. A school-age child is hit in the face with a baseball bat and reports pain in one eye. The primary care pediatric nurse practitioner is able to see a dark red fluid level between the cornea and iris on gross examination, but the child resists any exam with a light. Which action is correct? a. Administer an oral analgesic medication. b. Apply a Fox shield and reevaluate the eye in 24 hours. c. Instill anesthetic eyedrops into the affected eye. d. Refer the child immediately to an ophthalmologist. - Correct answer ANS: D This child has a traumatic injury with hyphema to the eye, and an ophthalmologist must examine the eye to rule out orbital hematoma or retinal detachment. Any further attempt to examine the child may result in further injury. A Fox shield is used once more serious injury is excluded. 4. During a well-baby assessment on a 1-week-old infant who had a normal exam when discharged from the newborn nursery 2 days prior, the primary care pediatric nurse practitioner notes moderate eyelid swelling, bulbar conjunctival injections, and moderate amounts of thick, purulent discharge. What is the likely diagnosis? a. Chemical-induced conjunctivitis b. Chlamydia trachomatis conjunctivitis c. Herpes simplex virus (HSV) conjunctivitis d. Neisseria gonorrhea conjunctivitis - Correct answer ANS: B C. trachomatis conjunctivitis usually begins between 5 to 14 days of life and causes moderate eyelid swelling, palpebral or bulbar conjunctivitis, and moderate, thick, purulent discharge. Chemical- induced conjunctivitis manifests as nonpurulent discharge. HSV is characterized by serosanguinous discharge. N. gonorrhea causes acute conjunctival inflammation and excessive purulent discharge. 5. The primary care pediatric nurse practitioner applies fluorescein stain to a child's eye. When examining the eye with a cobalt blue filter light, the entire cornea appears cloudy. What does this indicate? a. The cornea has not been damaged. b. There is too little stain on the cornea. c. There is damage to the cornea. d. There is too much stain on the cornea. - Correct answer ANS: D When fluorescein stain is applied and the entire cornea appears cloudy, it means that there is too much of the stain. Damaged areas of the cornea should appear greenish after staining with fluorescein dye. 6. During a well child assessment of an African-American infant, the primary care pediatric nurse practitioner notes a dark red-brown light reflex in the left eye and a slightly brighter, red-orange light reflex in the right eye. The nurse practitioner will P a g e 90 | 134 a. dilate the pupils and reassess the red reflex. b. order auto-refractor screening of the eyes. c. recheck the red reflex in 1 month. d. refer the infant to an ophthalmologist. - Correct answer ANS: D Any asymmetry, dark or white spots, opacities, or leukokoria should be referred immediately to a pediatric ophthalmologist. The PNP does not dilate pupils or order auto-refractor exams; these are done by an ophthalmologist. Because retinoblastoma is a concern, any unusual finding should be immediately referred. 7. The primary care pediatric nurse practitioner performs a Hirschberg test to evaluate a. color vision. b. ocular alignment. c. peripheral vision. d. visual acuity. - Correct answer ANS: B The Hirschberg test, or corneal light reflex, assesses ocular mobility and alignment by looking for symmetry of reflected light. Color vision testing is performed with Richmond pseudo- isochromatic plates. Peripheral vision is tested by watching the child's response to objects as they are moved in and out of the visual fields. Visual acuity is performed using eye charts or visual-evoked potential readings. 8. The primary care pediatric nurse practitioner performs a well baby assessment of a 5-day-old infant and notes mild conjunctivitis, corneal opacity, and serosanguinous discharge in the right eye. Which course of action is correct? a. Administer intramuscular ceftriaxone 50 mg/kg. b. Admit the infant to the hospital immediately. c. Give oral erythromycin 30 to 50 mg/kg/day for 2 weeks. d. Teach the parent how to perform tear duct massage. - Correct answer ANS: B The infant has symptoms consistent with HPV conjunctivitis and requires hospitalization for topical and systemic antiviral medications to prevent spread to the central nervous system, mouth, and skin. IM ceftriaxone is given for gonococcal conjunctivitis. Oral erythromycin is given for chlamydial conjunctivitis. Tear duct massage is performed for lacrimal duct obstruction. 9. The primary care pediatric nurse practitioner performs a vision screen on a 4-month-old infant and notes the presence of convergence and accommodation with mild esotropia of the left eye. What will the nurse practitioner do? a. Patch the right eye to improve coordination of the left eye. b. Reassure the parents that the infant will outgrow this. c. Recheck the infant's eyes in 2 to 4 weeks. d. Refer the infant to a pediatric ophthalmologist. - Correct answer ANS: D Esotropia that continues or occurs at 3 to 4 months of age is abnormal, so the infant should be referred to a pediatric ophthalmologist. The PNP does not determine whether an eye patch should be used. Because it is abnormal at this age, the PNP will not reassure the parents that the infant will outgrow this. Esotropia after 3 to 4 months of age must be evaluated by a specialist and not reevaluated in 2 to 4 weeks. 10. A toddler exhibits exotropia of the right eye during a cover-uncover screen. The primary care pediatric nurse practitioner will refer to a pediatric ophthalmologist to initiate which treatment? a. Botulinum toxin injection b. Corrective lenses c. Occluding the affected eye for 6 hours per day d. Patching of the unaffected eye for 2 hours each day - Correct answer ANS: D Deviations are initially treated by patching the unaffected eye for 2 hours each day to force the affected P a g e 91 | 134 eye to move correctly. Botulinum toxin injection may be used with some deviations but is not a first-line therapy. Corrective lenses alone improve amblyopia in 27% of patients. The unaffected eye is patched; 2 hours per day is as effective as 6 hours per day. 11. A preschool-age child is seen in the clinic after waking up a temperature of 102.2°F, swelling and erythema of the upper lid of one eye, and moderate pain when looking from side to side. Which course of treatment is correct? a. Admit to the hospital for intravenous antibiotics. b. Obtain a lumbar puncture and blood culture. c. Order warm compresses 4 times daily for 5 days. d. Prescribe a 10- to 14-day course of oral antibiotics. - Correct answer ANS: A This child has periorbital cellulitis and must be hospitalized because of having pain with movement of the eye, indicating orbital involvement. LP is performed on infants under 1 year of age. Warm compresses are used for mild cases. Oral antibiotics are not indicated. 12. A preschool-age child who attends day care has a 2-day history of matted eyelids in the morning and burning and itching of the eyes. The primary care pediatric nurse practitioner notes yellow-green purulent discharge from both eyes, conjunctival erythema, and mild URI symptoms. Which action is correct? a. Culture the conjunctival discharge. b. Observe the child for several days. c. Order an oral antibiotic medication. d. Prescribe topical antibiotic drops. - Correct answer ANS: D Young children with bacterial conjunctivitis may be treated with topical antibiotic drops. Culturing the eyes is not necessary unless there is no improvement. While most cases of bacterial conjunctivitis are self-limiting, using a topical antibiotic will hasten the return to day care. Oral antibiotics are not indicated. 13. The primary care pediatric nurse practitioner observes a tender, swollen red furuncle on the upper lid margin of a child's eye. What treatment will the nurse practitioner recommend? a. Culture of the lesion to determine causative organism b. Referral to ophthalmology for incision and drainage c. Topical steroid medication d. Warm, moist compresses 3 to 4 times daily - Correct answer ANS: D The child has symptoms of hordeolum, or stye. Although these often rupture spontaneously, warm, moist compresses may hasten this process. It is not necessary to culture the lesion unless symptoms do not resolve. Referral to ophthalmology is made if the hordeolum does not rupture on its own. Steroids are not indicated. 14. A school-age child is seen in the clinic after a fragment from a glass bottle flew into the eye. What will the primary care pediatric nurse practitioner do? a. Refer immediately to an ophthalmologist. b. Attempt to visualize the glass fragment. c. Irrigate the eye with sterile saline. d. Instill a topical anesthetic. - Correct answer ANS: A The PNP should never attempt to remove an intraocular foreign body or any projectile object but should refer immediately to an ophthalmologist. Visualizing the object, irrigating the eye, or instilling drops may further injure the eye. 15. A 14-year-old child has a 2-week history of severe itching and tearing of both eyes. The primary care pediatric nurse practitioner notes redness and swelling of the eyelids along with stringy, mucoid P a g e 92 | 134 d. The tubes will most likely remain in place for 3 to 4 years. - Correct answer ANS: A By reducing middle ear fluid, the child with hearing loss from this condition should show improvement in hearing. Children may still have infections but without persistent effusion. Earplugs are not necessary unless the child's head is submerged. PETs usually fall out on their own; if they are still in place 2 to 3 years after placement, they should be removed by the otolaryngology surgeon. 8. The parent of a 4-month-old infant is concerned that the infant cannot hear. Which test will the primary care pediatric nurse practitioner order to evaluate potential hearing loss in this infant? a. Acoustic reflectometry b. Audiometry c. Auditory brainstem response (ABR) d. Evoked otooacoustic emission (EOAE) testing - Correct answer ANS: C ABR is not a direct measure of hearing but allows for inferences to be made about hearing thresholds and is useful for identifying hearing loss in a young infant. Although sedation is occasionally required, this test is useful in infants and young children unable to cooperate with EOAE or audiometry. Acoustic reflectometry is used to detect middle ear effusion. Audiometry requires a cooperative child. EOAE is used for universal screening in newborns. The American Academy of Pediatrics (AAP) Bright Futures guidelines (AAP, 2014) recommends pure-tone audiometry at 3, 4, 5, 6, 8, 10, 12, 15, and 18 years of age. 9. A child with a history of otitis externa asks about ways to prevent this condition. What will the primary care pediatric nurse practitioner recommend? a. Cleaning ear canals well after swimming b. Drying the ear canal with a hair dryer c. Swimming only in chlorinated pools d. Using cerumenolytic agents daily - Correct answer ANS: B Otitis externa is most frequently caused by retained moisture in the ear canal after swimming and when the protective barriers on the skin break down. Drying the ear canals with a hair dryer on a low setting helps to remove the moisture. Cleaning the ear canals, swimming in chlorinated water, and using a cerumenolytic remove the wax that protects the ear canal from superficial infection. 10. A 3-year-old child has had one episode of acute otitis media 3 weeks prior with a normal tympanogram just after treatment with amoxicillin. In the clinic today, the child has a type B tympanogram, a temperature of 102.5°F, and a bulging tympanic membrane. What will the primary care pediatric nurse practitioner order? a. A referral for tympanocentesis b. Amoxicillin twice daily c. Amoxicillin-clavulanate twice daily d. Intramuscular ceftriaxone - Correct answer ANS: C Amoxicillin-clavulanate should be given for failed therapy with amoxicillin or when the child has had AOM treated with amoxicillin within the past month. 11. A 3-year-old child with pressure-equalizing tubes (PET) in both ears has otalgia in one ear. The primary care pediatric nurse practitioner is able to visualize the tube and does not see exudate in the ear canal and obtains a type A tympanogram. What will the nurse practitioner do? a. Order ototopical antibiotic/corticosteroid drops. b. Prescribe a prophylactic antibiotic medication. c. Reassure the parent that this is a normal exam. d. Refer the child to an otolaryngologist for follow-up - Correct answer ANS: A A normal, or type A, tympanogram in a child with PET may indicate a clogged tube. Ototopical antibiotic/corticosteroid drops can occasionally clear a clogged PET. Prophylactic antibiotics are not P a g e 95 | 134 recommended to prevent otitis media. It is not necessary to refer unless the pain continues in spite of standard measures. 12. The primary care pediatric nurse practitioner obtains a tympanogram on a child that reveals a sharp peak of -180 mm H2O. What does this value indicate? a. A normal tympanic membrane b. Middle ear effusion c. Negative ear pressure d. Tympanic membrane perforation - Correct answer ANS: C The type C tympanogram has a sharp peak between -100 and -200 mm H2O and reflects negative ear pressure. A normal tympanogram has a sharp positive peak or a type A tympanogram. Middle ear effusion and a TM perforation both cause a type B tympanogram with either no peak or a flattened wave. 13. A child complains of itching in both ears and is having trouble hearing. The primary care pediatric nurse practitioner notes periauricular edema and marked swelling of the external auditory canal and elicits severe pain when manipulating the external ear structures. Which is an appropriate intervention? a. Obtain a culture of the external auditory canal. b. Order ototopical antibiotic/corticosteroid drops. c. Prescribe oral amoxicillin-clavulanate. d. Refer the child to an otolaryngologist. - Correct answer ANS: B Ototopical antibiotic/corticosteroid drops are the mainstay of therapy for OE. It is not necessary to obtain a culture unless the infection does not respond to treatment. Oral antibiotics are not indicated unless impetigo occurs and is severe. A referral to a specialist is not recommended. 14. The primary care pediatric nurse practitioner notes a small, round object in a child's external auditory canal, near the tympanic membrane. The child's parent thinks it is probably a dried pea. What will the nurse practitioner do to remove this object? a. Irrigate the external auditory canal to flush out the object. b. Refer the child to an otolaryngologist for removal. c. Remove the object with a wire loop curette. d. Use a bayonet forceps to grasp and remove the object. - Correct answer ANS: B Spherical objects are the most difficult to remove and should be referred. Irrigation is not recommended for objects made of organic material and also increases the risk of pushing the object farther down. 1. A 5-year-old child who had a repair for transposition of the great arteries shortly after birth is growing normally and has been asymptomatic since the surgery. The primary care nurse practitioner notes mild shortness of breath with exertion and, upon questioning, learns that the child has recently complained of dizziness. What will the nurse practitioner do? a. Order an echocardiogram and chest radiograph. b. Perform pulmonary function testing. c. Reassure the parent that these symptoms are common. d. Refer the child to the cardiologist immediately. - Correct answer ANS: D Children with a history of d-TGA who have a history of palpitations, syncope, or shortness of breath should be referred to a cardiologist. Echocardiograms should be performed annually under the supervision of the cardiologist. Pulmonary function testing is not indicated. These symptoms may represent problems in patency with the coronary arteries and are not common. P a g e 96 | 134 2. A 12-year-old child whose weight and BMI are in the 75th percentile has a diastolic blood pressure that is between the 95th and 99th percentiles for age, sex, and height on three separate occasions. Initial tests for this child will include a. complete blood count. b. erythrocyte sedimentation rate. c. renal function and plasma renin tests. d. urinalysis and electrolytes. - Correct answer ANS: C Since the majority of children with stage 1 or 2 hypertension have renal or renovascular causes for elevated BP, renal function and plasma renin tests should be performed. Children under 10 years of age with stage 2 hypertension should have more aggressive laboratory evaluation, including CBC, ESR, UA, and electrolytes. 3. During a well baby examination of a 6-week-old infant, the primary care pediatric nurse practitioner notes poor weight gain, acrocyanosis of the hands and feet, and a respiratory rate of 60 breaths per minute. Oxygen saturation on room air is 93%. The remainder of the exam is unremarkable. Which action is correct? a. Follow-up in 1 week to assess the infant's weight. b. Order a chest radiograph and an electrocardiogram. c. Reassure the parents that the exam is within normal limits. d. Refer the infant to a pediatric cardiologist. - Correct answer ANS: D Infants with oxygen saturation less than 95% and those with poor feeding should be referred emergently to a cardiologist. The infant may have CHF and will need to be evaluated. 4. The primary care pediatric nurse practitioner is performing a well child examination on a school-age child who had complete repair of a tetralogy of Fallot defect in infancy. What is important in this child's health maintenance regime? a. Cardiology clearance for sports participation b. Restriction of physical activity to avoid pulmonary complications c. Sub-acute bacterial endocarditis prophylaxis precautions d. Teaching about management of hypercyanotic episodes - Correct answer ANS: A Children who have had TOF repair must be cleared by cardiology before participation in sports, but there is no need to restrict all physical activity. SBE prophylaxis is given prior to surgery and for 6 months afterward. Hypercyanotic episodes occur before repair. 5. The primary care pediatric nurse practitioner performs a well child examination on a 12- month-old child who had repair of a congenital heart defect at 8 months of age. The child has a normal exam. The parent reports that the child is not taking any medications. The nurse practitioner will contact the child's cardiologist to discuss whether the child needs which medication? a. Amoxicillin b. Capoten c. Digoxin d. Furosemide - Correct answer ANS: A Children who have had complete repair of CHD should have SBE prophylaxis with amoxicillin for 6 months after the procedure. Capoten, an antihypertensive, digoxin, an inotropic medication, and furosemide, a diuretic, are given for specific symptoms as indicated. 6. An infant with trisomy 21 has a complete AV canal defect. Which finding, associated with having both of these conditions, will the primary care pediatric nurse practitioner expect? a. Crackles in both lungs b. Hepatomegaly P a g e 97 | 134
Docsity logo



Copyright © 2024 Ladybird Srl - Via Leonardo da Vinci 16, 10126, Torino, Italy - VAT 10816460017 - All rights reserved